How a Simple Urine Test Could Reveal Early-Stage Lung Cancer

Article Type
Changed
Fri, 01/19/2024 - 14:23

Lung cancer is the deadliest cancer in the world, largely because so many patients are diagnosed late.

Screening more patients could help, yet screening rates remain critically low. In the United States, only about 6% of eligible people get screened , according to the American Lung Association. Contrast that with screening rates for breast, cervical, and colorectal cancer, which all top 70%.

But what if lung cancer detection was as simple as taking a puff on an inhaler and following up with a urine test?

Researchers at the Massachusetts Institute of Technology (MIT), Cambridge, Massachusetts, have developed nanosensors that target lung cancer proteins and can be delivered via inhaler or nebulizer, according to research published this month in Science Advances. If the sensors spot these proteins, they produce a signal in the urine that can be detected with a paper test strip.

“It’s a more complex version of a pregnancy test, but it’s very simple to use,” said Qian Zhong, PhD, an MIT researcher and co-lead author of the study.

Currently, the only recommended screening test for lung cancer is low-dose CT. But not everyone has easy access to screening facilities, said the other co-lead author Edward Tan, PhD, a former MIT postdoc and currently a scientist at the biotech company Prime Medicine, Cambridge, Massachusetts.

“Our focus is to provide an alternative for the early detection of lung cancer that does not rely on resource-intensive infrastructure,” said Dr. Tan. “Most developing countries don’t have such resources” — and residents in some parts of the United States don’t have easy access, either, he said.
 

How It Works

The sensors are polymer nanoparticles coated in DNA barcodes, short DNA sequences that are unique and easy to identify. The researchers engineered the particles to be targeted by protease enzymes linked to stage I lung adenocarcinoma. Upon contact, the proteases cleave off the barcodes, which make their way into the bloodstream and are excreted in urine. A test strip can detect them, revealing results about 20 minutes from the time it’s dipped.

The researchers tested this system in mice genetically engineered to develop human-like lung tumors. Using aerosol nebulizers, they delivered 20 sensors to mice with the equivalent of stage I or II cancer. Using a machine learning algorithm, they identified the four most accurate sensors. With 100% specificity, those four sensors exhibited sensitivity of 84.6%.

“One advantage of using inhalation is that it’s noninvasive, and another advantage is that it distributes across the lung quite homogeneously,” said Dr. Tan. The time from inhalation to detection is also relatively fast — in mice, the whole process took about 2 hours, and Dr. Zhong speculated that it would not be much longer in humans.
 

Other Applications and Challenges

An injectable version of this technology, also developed at MIT, has already been tested in a phase 1 clinical trial for diagnosing liver cancer and nonalcoholic steatohepatitis. The injection also works in tandem with a urine test, the researchers showed in 2021. According to Tan, his research group (led by  Sangeeta Bhatia, MD, PhD) was the first to describe this type of technology to screen for diseases.

The lab is also working toward using inhalable sensors to distinguish between viral, bacterial, and fungal pneumonia. And the technology could also be used to diagnose other lung conditions like asthma and chronic obstructive pulmonary disease, Dr. Tan said.

The tech is certainly “innovative,” remarked Gaetano Rocco, MD, a thoracic surgeon and lung cancer researcher at Memorial Sloan Kettering Cancer Center, Basking Ridge, New Jersey, who was not involved in the study.

Still, challenges may arise when applying it to people. Many factors are involved in regulating fluid volume, potentially interfering with the ability to detect the compounds in the urine, Rocco said. Diet, hydration, drug interference, renal function, and some chronic diseases could all limit effectiveness.

Another challenge: Human cancer can be more heterogeneous (containing different kinds of cancer cells), so four sensors may not be enough, Zhong said. He and colleagues are beginning to analyze human biopsy samples to see whether the same sensors that worked in mice would also work in humans. If all goes well, they hope to do studies on humans or nonhuman primates.
 

A version of this article appeared on Medscape.com.

Publications
Topics
Sections

Lung cancer is the deadliest cancer in the world, largely because so many patients are diagnosed late.

Screening more patients could help, yet screening rates remain critically low. In the United States, only about 6% of eligible people get screened , according to the American Lung Association. Contrast that with screening rates for breast, cervical, and colorectal cancer, which all top 70%.

But what if lung cancer detection was as simple as taking a puff on an inhaler and following up with a urine test?

Researchers at the Massachusetts Institute of Technology (MIT), Cambridge, Massachusetts, have developed nanosensors that target lung cancer proteins and can be delivered via inhaler or nebulizer, according to research published this month in Science Advances. If the sensors spot these proteins, they produce a signal in the urine that can be detected with a paper test strip.

“It’s a more complex version of a pregnancy test, but it’s very simple to use,” said Qian Zhong, PhD, an MIT researcher and co-lead author of the study.

Currently, the only recommended screening test for lung cancer is low-dose CT. But not everyone has easy access to screening facilities, said the other co-lead author Edward Tan, PhD, a former MIT postdoc and currently a scientist at the biotech company Prime Medicine, Cambridge, Massachusetts.

“Our focus is to provide an alternative for the early detection of lung cancer that does not rely on resource-intensive infrastructure,” said Dr. Tan. “Most developing countries don’t have such resources” — and residents in some parts of the United States don’t have easy access, either, he said.
 

How It Works

The sensors are polymer nanoparticles coated in DNA barcodes, short DNA sequences that are unique and easy to identify. The researchers engineered the particles to be targeted by protease enzymes linked to stage I lung adenocarcinoma. Upon contact, the proteases cleave off the barcodes, which make their way into the bloodstream and are excreted in urine. A test strip can detect them, revealing results about 20 minutes from the time it’s dipped.

The researchers tested this system in mice genetically engineered to develop human-like lung tumors. Using aerosol nebulizers, they delivered 20 sensors to mice with the equivalent of stage I or II cancer. Using a machine learning algorithm, they identified the four most accurate sensors. With 100% specificity, those four sensors exhibited sensitivity of 84.6%.

“One advantage of using inhalation is that it’s noninvasive, and another advantage is that it distributes across the lung quite homogeneously,” said Dr. Tan. The time from inhalation to detection is also relatively fast — in mice, the whole process took about 2 hours, and Dr. Zhong speculated that it would not be much longer in humans.
 

Other Applications and Challenges

An injectable version of this technology, also developed at MIT, has already been tested in a phase 1 clinical trial for diagnosing liver cancer and nonalcoholic steatohepatitis. The injection also works in tandem with a urine test, the researchers showed in 2021. According to Tan, his research group (led by  Sangeeta Bhatia, MD, PhD) was the first to describe this type of technology to screen for diseases.

The lab is also working toward using inhalable sensors to distinguish between viral, bacterial, and fungal pneumonia. And the technology could also be used to diagnose other lung conditions like asthma and chronic obstructive pulmonary disease, Dr. Tan said.

The tech is certainly “innovative,” remarked Gaetano Rocco, MD, a thoracic surgeon and lung cancer researcher at Memorial Sloan Kettering Cancer Center, Basking Ridge, New Jersey, who was not involved in the study.

Still, challenges may arise when applying it to people. Many factors are involved in regulating fluid volume, potentially interfering with the ability to detect the compounds in the urine, Rocco said. Diet, hydration, drug interference, renal function, and some chronic diseases could all limit effectiveness.

Another challenge: Human cancer can be more heterogeneous (containing different kinds of cancer cells), so four sensors may not be enough, Zhong said. He and colleagues are beginning to analyze human biopsy samples to see whether the same sensors that worked in mice would also work in humans. If all goes well, they hope to do studies on humans or nonhuman primates.
 

A version of this article appeared on Medscape.com.

Lung cancer is the deadliest cancer in the world, largely because so many patients are diagnosed late.

Screening more patients could help, yet screening rates remain critically low. In the United States, only about 6% of eligible people get screened , according to the American Lung Association. Contrast that with screening rates for breast, cervical, and colorectal cancer, which all top 70%.

But what if lung cancer detection was as simple as taking a puff on an inhaler and following up with a urine test?

Researchers at the Massachusetts Institute of Technology (MIT), Cambridge, Massachusetts, have developed nanosensors that target lung cancer proteins and can be delivered via inhaler or nebulizer, according to research published this month in Science Advances. If the sensors spot these proteins, they produce a signal in the urine that can be detected with a paper test strip.

“It’s a more complex version of a pregnancy test, but it’s very simple to use,” said Qian Zhong, PhD, an MIT researcher and co-lead author of the study.

Currently, the only recommended screening test for lung cancer is low-dose CT. But not everyone has easy access to screening facilities, said the other co-lead author Edward Tan, PhD, a former MIT postdoc and currently a scientist at the biotech company Prime Medicine, Cambridge, Massachusetts.

“Our focus is to provide an alternative for the early detection of lung cancer that does not rely on resource-intensive infrastructure,” said Dr. Tan. “Most developing countries don’t have such resources” — and residents in some parts of the United States don’t have easy access, either, he said.
 

How It Works

The sensors are polymer nanoparticles coated in DNA barcodes, short DNA sequences that are unique and easy to identify. The researchers engineered the particles to be targeted by protease enzymes linked to stage I lung adenocarcinoma. Upon contact, the proteases cleave off the barcodes, which make their way into the bloodstream and are excreted in urine. A test strip can detect them, revealing results about 20 minutes from the time it’s dipped.

The researchers tested this system in mice genetically engineered to develop human-like lung tumors. Using aerosol nebulizers, they delivered 20 sensors to mice with the equivalent of stage I or II cancer. Using a machine learning algorithm, they identified the four most accurate sensors. With 100% specificity, those four sensors exhibited sensitivity of 84.6%.

“One advantage of using inhalation is that it’s noninvasive, and another advantage is that it distributes across the lung quite homogeneously,” said Dr. Tan. The time from inhalation to detection is also relatively fast — in mice, the whole process took about 2 hours, and Dr. Zhong speculated that it would not be much longer in humans.
 

Other Applications and Challenges

An injectable version of this technology, also developed at MIT, has already been tested in a phase 1 clinical trial for diagnosing liver cancer and nonalcoholic steatohepatitis. The injection also works in tandem with a urine test, the researchers showed in 2021. According to Tan, his research group (led by  Sangeeta Bhatia, MD, PhD) was the first to describe this type of technology to screen for diseases.

The lab is also working toward using inhalable sensors to distinguish between viral, bacterial, and fungal pneumonia. And the technology could also be used to diagnose other lung conditions like asthma and chronic obstructive pulmonary disease, Dr. Tan said.

The tech is certainly “innovative,” remarked Gaetano Rocco, MD, a thoracic surgeon and lung cancer researcher at Memorial Sloan Kettering Cancer Center, Basking Ridge, New Jersey, who was not involved in the study.

Still, challenges may arise when applying it to people. Many factors are involved in regulating fluid volume, potentially interfering with the ability to detect the compounds in the urine, Rocco said. Diet, hydration, drug interference, renal function, and some chronic diseases could all limit effectiveness.

Another challenge: Human cancer can be more heterogeneous (containing different kinds of cancer cells), so four sensors may not be enough, Zhong said. He and colleagues are beginning to analyze human biopsy samples to see whether the same sensors that worked in mice would also work in humans. If all goes well, they hope to do studies on humans or nonhuman primates.
 

A version of this article appeared on Medscape.com.

Publications
Publications
Topics
Article Type
Sections
Disallow All Ads
Content Gating
No Gating (article Unlocked/Free)
Alternative CME
Disqus Comments
Default
Use ProPublica
Hide sidebar & use full width
render the right sidebar.
Conference Recap Checkbox
Not Conference Recap
Clinical Edge
Display the Slideshow in this Article
Medscape Article
Display survey writer
Reuters content
Disable Inline Native ads
WebMD Article

Invasive Procedures and Complications Follow Lung Cancer Screening

Article Type
Changed
Fri, 01/19/2024 - 14:33

 

TOPLINE:

After lung cancer screening (LCS), imaging, and invasive procedures were performed 31.9% and 2.8% of the time, respectively. Complications during invasive procedures occurred in 30.6% of cases. 

METHODOLOGY:

  • Researchers analyzed data from 9266 patients aged 55-80 years who completed at least one LCS with low-dose CT (LDCT) between 2014 and 2018.
  • This study used data from the PROSPR Lung Consortium.
  • Results were compared with findings from the National Lung Screening Trial (NLST), a large study of smokers published in 2011.

TAKEAWAY:

  • In total, 2956 patients (31.9%) underwent follow-up imaging, including CT, LDCT, MRI, or PET; 180 (0.02%) had invasive procedures, including needle biopsy, bronchoscopy, mediastinoscopy or mediastinotomy, or thoracoscopy.
  • Within 30 days after an invasive diagnostic procedure, 55 of 180 patients (30.6%) experienced complications; 20.6% were major, 8.3% were intermediate, and 1.7% were minor.
  • Complication rates after invasive procedures were higher in PROSPR than the NLST (30.6% vs 17.7%).
  • Compared with all patients, those with an abnormal LCS were slightly older, more likely to currently smoke, reported more packs of cigarettes smoked daily, and had more comorbid conditions.
  • In 2013, the US Preventive Services Task Force recommended annual LCS for certain people who smoke, on the basis of findings from the NLST.

IN PRACTICE:

“We observed higher rates of both invasive procedures and complications than those observed in NLST, highlighting the need for practice-based strategies to assess variations in the quality of care and to prioritize LCS among those patients most likely to receive a net benefit from screening in relation to potential complications and other harms,” the researchers wrote. 

SOURCE:

Katharine A. Rendle, PhD, MSW, MPH, with Perelman School of Medicine, University of Pennsylvania, is the study’s corresponding author. The study was published online in Annals of Internal Medicine.

LIMITATIONS:

This study was retrospective, and data were analyzed using procedural coding. In addition, the NLST based abnormal findings on different criteria from those used in clinical practice (Lung-RADS), making direct comparison of patients difficult. Patients in PROSPR were older, more likely to be currently smoking, and had higher rates of comorbid conditions compared with patients in the NLST. 

DISCLOSURES:

This study was supported by grants from the National Cancer Institute and the Gordon and Betty Moore Foundation.

Publications
Topics
Sections

 

TOPLINE:

After lung cancer screening (LCS), imaging, and invasive procedures were performed 31.9% and 2.8% of the time, respectively. Complications during invasive procedures occurred in 30.6% of cases. 

METHODOLOGY:

  • Researchers analyzed data from 9266 patients aged 55-80 years who completed at least one LCS with low-dose CT (LDCT) between 2014 and 2018.
  • This study used data from the PROSPR Lung Consortium.
  • Results were compared with findings from the National Lung Screening Trial (NLST), a large study of smokers published in 2011.

TAKEAWAY:

  • In total, 2956 patients (31.9%) underwent follow-up imaging, including CT, LDCT, MRI, or PET; 180 (0.02%) had invasive procedures, including needle biopsy, bronchoscopy, mediastinoscopy or mediastinotomy, or thoracoscopy.
  • Within 30 days after an invasive diagnostic procedure, 55 of 180 patients (30.6%) experienced complications; 20.6% were major, 8.3% were intermediate, and 1.7% were minor.
  • Complication rates after invasive procedures were higher in PROSPR than the NLST (30.6% vs 17.7%).
  • Compared with all patients, those with an abnormal LCS were slightly older, more likely to currently smoke, reported more packs of cigarettes smoked daily, and had more comorbid conditions.
  • In 2013, the US Preventive Services Task Force recommended annual LCS for certain people who smoke, on the basis of findings from the NLST.

IN PRACTICE:

“We observed higher rates of both invasive procedures and complications than those observed in NLST, highlighting the need for practice-based strategies to assess variations in the quality of care and to prioritize LCS among those patients most likely to receive a net benefit from screening in relation to potential complications and other harms,” the researchers wrote. 

SOURCE:

Katharine A. Rendle, PhD, MSW, MPH, with Perelman School of Medicine, University of Pennsylvania, is the study’s corresponding author. The study was published online in Annals of Internal Medicine.

LIMITATIONS:

This study was retrospective, and data were analyzed using procedural coding. In addition, the NLST based abnormal findings on different criteria from those used in clinical practice (Lung-RADS), making direct comparison of patients difficult. Patients in PROSPR were older, more likely to be currently smoking, and had higher rates of comorbid conditions compared with patients in the NLST. 

DISCLOSURES:

This study was supported by grants from the National Cancer Institute and the Gordon and Betty Moore Foundation.

 

TOPLINE:

After lung cancer screening (LCS), imaging, and invasive procedures were performed 31.9% and 2.8% of the time, respectively. Complications during invasive procedures occurred in 30.6% of cases. 

METHODOLOGY:

  • Researchers analyzed data from 9266 patients aged 55-80 years who completed at least one LCS with low-dose CT (LDCT) between 2014 and 2018.
  • This study used data from the PROSPR Lung Consortium.
  • Results were compared with findings from the National Lung Screening Trial (NLST), a large study of smokers published in 2011.

TAKEAWAY:

  • In total, 2956 patients (31.9%) underwent follow-up imaging, including CT, LDCT, MRI, or PET; 180 (0.02%) had invasive procedures, including needle biopsy, bronchoscopy, mediastinoscopy or mediastinotomy, or thoracoscopy.
  • Within 30 days after an invasive diagnostic procedure, 55 of 180 patients (30.6%) experienced complications; 20.6% were major, 8.3% were intermediate, and 1.7% were minor.
  • Complication rates after invasive procedures were higher in PROSPR than the NLST (30.6% vs 17.7%).
  • Compared with all patients, those with an abnormal LCS were slightly older, more likely to currently smoke, reported more packs of cigarettes smoked daily, and had more comorbid conditions.
  • In 2013, the US Preventive Services Task Force recommended annual LCS for certain people who smoke, on the basis of findings from the NLST.

IN PRACTICE:

“We observed higher rates of both invasive procedures and complications than those observed in NLST, highlighting the need for practice-based strategies to assess variations in the quality of care and to prioritize LCS among those patients most likely to receive a net benefit from screening in relation to potential complications and other harms,” the researchers wrote. 

SOURCE:

Katharine A. Rendle, PhD, MSW, MPH, with Perelman School of Medicine, University of Pennsylvania, is the study’s corresponding author. The study was published online in Annals of Internal Medicine.

LIMITATIONS:

This study was retrospective, and data were analyzed using procedural coding. In addition, the NLST based abnormal findings on different criteria from those used in clinical practice (Lung-RADS), making direct comparison of patients difficult. Patients in PROSPR were older, more likely to be currently smoking, and had higher rates of comorbid conditions compared with patients in the NLST. 

DISCLOSURES:

This study was supported by grants from the National Cancer Institute and the Gordon and Betty Moore Foundation.

Publications
Publications
Topics
Article Type
Sections
Disallow All Ads
Content Gating
No Gating (article Unlocked/Free)
Alternative CME
Disqus Comments
Default
Use ProPublica
Hide sidebar & use full width
render the right sidebar.
Conference Recap Checkbox
Not Conference Recap
Clinical Edge
Display the Slideshow in this Article
Medscape Article
Display survey writer
Reuters content
Disable Inline Native ads
WebMD Article

How scientists are uncovering the mysteries of ARDS

Article Type
Changed
Fri, 01/05/2024 - 00:15

Scientists are beginning to unravel the secrets of acute respiratory distress syndrome (ARDS), the devastating disorder that floods the lungs with fluid and has ushered countless millions to death after infection with pneumonia, sepsis, and COVID-19.

Two centuries after the lung damage caused by the disorder was first described in medicine, it’s now clear that ARDS is an autoimmune condition spurred by the body’s overactive defenses. There’s interest in disrupting “crosstalk” between cells, and rise of a new form of genetic analysis is allowing researchers to test their hypotheses more effectively than ever before. And, perhaps most importantly, recent findings reveal how stem cells in the epithelial lining of the lungs get stalled in an intermediate stage before regenerating into new cells. Reversing this process could trigger repair and recovery.

There’s still a ways to go before clinical trials can test therapies to turn things around at the epithelial level, acknowledged University of Michigan, Ann Arbor, professor of internal medicine Rachel L. Zemans, MD, in an interview. Still, “it’s a pretty exciting time,” said Dr Zemans, who manages a lab that explores how the lung epithelium responds to injury.
 

A lung disorder’s deep roots in human history

A British doctor first described the traits of ARDS in 1821, although this form of pulmonary edema had been described in “ancient writings,” according to a 2005 report by Gordon Bernard, MD, of Vanderbilt University Medical Center, Nashville, Tennessee. Sometimes called “double pneumonia,” ARDS was almost always fatal until the last few decades of the 20th century. “The advent of well-equipped ICUs, well-trained staff, and the availability of reliable positive pressure ventilators has allowed patients to be kept alive much longer and thus have the opportunity to heal the pulmonary injury and survive,” Dr Bernard wrote.

According to the Mayo Clinic, there are many causes of ARDS. Sepsis is the most common, and others include severe pneumonia, head/chest injuries, massive blood transfusions, pancreatitis, burns, and inhalation of harmful substances. Since 2020, ARDS has been a hallmark of COVID-19.

In an interview, University of Washington, Seattle, emeritus professor of medicine Thomas R. Martin MD, explained that ARDS occurs when the epithelium barrier in the lungs breaks down. Unlike the permeable endothelial barrier, the alveolar epithelium is “like a brick wall or a big dam, keeping red cells and plasma out of the airspace.”

In cases of pulmonary edema due to heart failure, fluid can back up into the lungs, said Dr Martin, who studies ARDS. However, pumps in the epithelium can clear that excess fluid pretty quickly because the epithelium remains in a normal state, he said. “Given enough time and enough medical support, people with heart failure and pulmonary edema can get better without lung injury.”

In ARDS, however, “the epithelium is damaged. Cells die in the alveolar wall, the scaffolding is exposed, and the fluid in the alveoli cannot be cleared out. You’ve got a disaster on your hands because all of the fluid and red blood cells and inflammatory products in the blood are going right into the airspace. The patient gets extremely short of breath because their oxygen level falls.”
 

 

 

COVID-19 virus finds a weak spot in the lungs

COVID-19 is “a classic example of an attack on the alveolar epithelium,” Dr Martin said.

By chance, the virus evolved to recognize receptors in the epithelium, allowing it to enter and propagate. “To make matters worse, defense mechanisms in the body attack those dying epithelial cells because the virus is visible on the surface cells. So lymphocytes from the immune system and macrophages attack the outer walls and cause further damage.”

Other scientists agree about this general picture of ARDS. “Studies of human lung tissue support the notion that failure of alveolar repair and regeneration mechanisms underlie the chronic lung dysfunction that can result from ARD,” wrote researchers from Cedars-Sinai Medical Center, Los Angeles, California, and Icahn School of Medicine at Mount Sinai, New York, NY, in a 2022 report.

According to Dr Martin, researchers and clinicians have discovered a pair of strategies to help vanquish COVID-19: Control viral entry through antiviral medication and dampen the body’s inflammatory response via steroids.

Still, “although we’ve learned lessons from COVID-19, we’re not good at all at promoting repair,” Dr Martin said. While new drugs have dramatically improved treatment for lung diseases such as cystic fibrosis, he said, “we don’t have good examples of new therapies that promote repair in ARDS.”
 

Looking for a way to turn the tide of fluid buildup

Dr Zemans and colleagues have uncovered a crucial obstacle to repair: the failure of stem cells to fully differentiate and become functional alveolar epithelial cells.

Researchers only began to understand a few years ago that the stem cells go through a transitional stage from type 2 to type 1, which make up 98% of cells in the epithelial surface, Dr Zemans said. In patients with ARDS who don’t get better quickly, “it looks like the cells get hung up in this intermediate state. They can’t finish that regeneration.”

As a 2022 study by Dr Zemans and colleagues put it, this process can lead to “ongoing barrier permeability, noncardiogenic pulmonary edema, and ventilator dependence, and mortality.” In fact, she said, “when we look at the lungs of people who died of ARDS, their cells were all in that intermediate stage.”

The discovery of the intermediate state only came about because of new technology called single-cell RNA sequencing, she said. “Now, these transitional cells are being found in other organs.”

Why do the epithelial cells get only part way through the regeneration process? It’s not entirely clear, Dr Zemans said, but researcher are intrigued by the idea that “cross-talk” between cells is playing a role.

“When the cells are in that stage, they also activate neighboring cells, including inflammatory cells, like macrophages, and fibroblasts,” she said. “And once those cells become activated, they become pathologic. What we think is that those cells then can talk back to the epithelial cells and prevent the epithelial cells from finishing that differentiation. It’s really hard to snap out of that positive feedback loop.”

This interaction probably evolved “for a good reason,” she said, “but it also became pathologic.” If the cells stay in the intermediate stage too long, she said, fibrosis develops. “They have scar tissue that never goes away. It takes a lot of work to expand the lungs when they’re so stiff when they should be stretchy like a rubber band. Scar tissue also gets in the way of the oxygen absorption, so some people have low oxygen levels.”
 

 

 

Future directions: Teaching cells to get “unstuck”

What’s next for research? One direction is exploring the variety of types of cells in the epithelium. Recent finding are revealing “new cell subpopulations that maintain alveolar homeostasis, communicate injury signals, and participate in normal and maladaptive repair. Emerging data illuminate the complexity of alveolar physiology and pathology to provide a more complete picture of how alveoli maintain health and respond to injurious stimuli,” write the researchers from Cedars-Sinai Medical Center and Icahn School of Medicine at Mount Sinai in their 2022 report.

Meanwhile, “we’re trying to look at the signaling pathways, the proteins or molecules, to understand the signals that tell a cell how to get unstuck,” Dr Zemans said. And researchers are exploring whether knocking out certain genes expressed by transitional cells in mice will lead to better outcomes, she said.

The 2022 study by Dr Zeman and colleagues described the potential ramifications of better understanding of the entire process: “Ultimately, investigation of the cellular and molecular mechanisms underlying ineffectual alveolar regeneration in ARDS and fibrosis may lead to novel therapies to promote physiological regeneration, thus accelerating restoration of barrier integrity, resolution of edema, liberation from the ventilator and survival in ARDS, and preventing fibrosis in fibroproliferative ARDS and [idiopathic pulmonary fibrosis].”

To put it more simply, “if you can seal the barrier, you can get the fluid out of the lungs, and you can get the patients off the ventilator, get out of the ICU, and go home,” Dr Zemans said.

Dr Zemans and Dr Martin have no disclosures.

Publications
Topics
Sections

Scientists are beginning to unravel the secrets of acute respiratory distress syndrome (ARDS), the devastating disorder that floods the lungs with fluid and has ushered countless millions to death after infection with pneumonia, sepsis, and COVID-19.

Two centuries after the lung damage caused by the disorder was first described in medicine, it’s now clear that ARDS is an autoimmune condition spurred by the body’s overactive defenses. There’s interest in disrupting “crosstalk” between cells, and rise of a new form of genetic analysis is allowing researchers to test their hypotheses more effectively than ever before. And, perhaps most importantly, recent findings reveal how stem cells in the epithelial lining of the lungs get stalled in an intermediate stage before regenerating into new cells. Reversing this process could trigger repair and recovery.

There’s still a ways to go before clinical trials can test therapies to turn things around at the epithelial level, acknowledged University of Michigan, Ann Arbor, professor of internal medicine Rachel L. Zemans, MD, in an interview. Still, “it’s a pretty exciting time,” said Dr Zemans, who manages a lab that explores how the lung epithelium responds to injury.
 

A lung disorder’s deep roots in human history

A British doctor first described the traits of ARDS in 1821, although this form of pulmonary edema had been described in “ancient writings,” according to a 2005 report by Gordon Bernard, MD, of Vanderbilt University Medical Center, Nashville, Tennessee. Sometimes called “double pneumonia,” ARDS was almost always fatal until the last few decades of the 20th century. “The advent of well-equipped ICUs, well-trained staff, and the availability of reliable positive pressure ventilators has allowed patients to be kept alive much longer and thus have the opportunity to heal the pulmonary injury and survive,” Dr Bernard wrote.

According to the Mayo Clinic, there are many causes of ARDS. Sepsis is the most common, and others include severe pneumonia, head/chest injuries, massive blood transfusions, pancreatitis, burns, and inhalation of harmful substances. Since 2020, ARDS has been a hallmark of COVID-19.

In an interview, University of Washington, Seattle, emeritus professor of medicine Thomas R. Martin MD, explained that ARDS occurs when the epithelium barrier in the lungs breaks down. Unlike the permeable endothelial barrier, the alveolar epithelium is “like a brick wall or a big dam, keeping red cells and plasma out of the airspace.”

In cases of pulmonary edema due to heart failure, fluid can back up into the lungs, said Dr Martin, who studies ARDS. However, pumps in the epithelium can clear that excess fluid pretty quickly because the epithelium remains in a normal state, he said. “Given enough time and enough medical support, people with heart failure and pulmonary edema can get better without lung injury.”

In ARDS, however, “the epithelium is damaged. Cells die in the alveolar wall, the scaffolding is exposed, and the fluid in the alveoli cannot be cleared out. You’ve got a disaster on your hands because all of the fluid and red blood cells and inflammatory products in the blood are going right into the airspace. The patient gets extremely short of breath because their oxygen level falls.”
 

 

 

COVID-19 virus finds a weak spot in the lungs

COVID-19 is “a classic example of an attack on the alveolar epithelium,” Dr Martin said.

By chance, the virus evolved to recognize receptors in the epithelium, allowing it to enter and propagate. “To make matters worse, defense mechanisms in the body attack those dying epithelial cells because the virus is visible on the surface cells. So lymphocytes from the immune system and macrophages attack the outer walls and cause further damage.”

Other scientists agree about this general picture of ARDS. “Studies of human lung tissue support the notion that failure of alveolar repair and regeneration mechanisms underlie the chronic lung dysfunction that can result from ARD,” wrote researchers from Cedars-Sinai Medical Center, Los Angeles, California, and Icahn School of Medicine at Mount Sinai, New York, NY, in a 2022 report.

According to Dr Martin, researchers and clinicians have discovered a pair of strategies to help vanquish COVID-19: Control viral entry through antiviral medication and dampen the body’s inflammatory response via steroids.

Still, “although we’ve learned lessons from COVID-19, we’re not good at all at promoting repair,” Dr Martin said. While new drugs have dramatically improved treatment for lung diseases such as cystic fibrosis, he said, “we don’t have good examples of new therapies that promote repair in ARDS.”
 

Looking for a way to turn the tide of fluid buildup

Dr Zemans and colleagues have uncovered a crucial obstacle to repair: the failure of stem cells to fully differentiate and become functional alveolar epithelial cells.

Researchers only began to understand a few years ago that the stem cells go through a transitional stage from type 2 to type 1, which make up 98% of cells in the epithelial surface, Dr Zemans said. In patients with ARDS who don’t get better quickly, “it looks like the cells get hung up in this intermediate state. They can’t finish that regeneration.”

As a 2022 study by Dr Zemans and colleagues put it, this process can lead to “ongoing barrier permeability, noncardiogenic pulmonary edema, and ventilator dependence, and mortality.” In fact, she said, “when we look at the lungs of people who died of ARDS, their cells were all in that intermediate stage.”

The discovery of the intermediate state only came about because of new technology called single-cell RNA sequencing, she said. “Now, these transitional cells are being found in other organs.”

Why do the epithelial cells get only part way through the regeneration process? It’s not entirely clear, Dr Zemans said, but researcher are intrigued by the idea that “cross-talk” between cells is playing a role.

“When the cells are in that stage, they also activate neighboring cells, including inflammatory cells, like macrophages, and fibroblasts,” she said. “And once those cells become activated, they become pathologic. What we think is that those cells then can talk back to the epithelial cells and prevent the epithelial cells from finishing that differentiation. It’s really hard to snap out of that positive feedback loop.”

This interaction probably evolved “for a good reason,” she said, “but it also became pathologic.” If the cells stay in the intermediate stage too long, she said, fibrosis develops. “They have scar tissue that never goes away. It takes a lot of work to expand the lungs when they’re so stiff when they should be stretchy like a rubber band. Scar tissue also gets in the way of the oxygen absorption, so some people have low oxygen levels.”
 

 

 

Future directions: Teaching cells to get “unstuck”

What’s next for research? One direction is exploring the variety of types of cells in the epithelium. Recent finding are revealing “new cell subpopulations that maintain alveolar homeostasis, communicate injury signals, and participate in normal and maladaptive repair. Emerging data illuminate the complexity of alveolar physiology and pathology to provide a more complete picture of how alveoli maintain health and respond to injurious stimuli,” write the researchers from Cedars-Sinai Medical Center and Icahn School of Medicine at Mount Sinai in their 2022 report.

Meanwhile, “we’re trying to look at the signaling pathways, the proteins or molecules, to understand the signals that tell a cell how to get unstuck,” Dr Zemans said. And researchers are exploring whether knocking out certain genes expressed by transitional cells in mice will lead to better outcomes, she said.

The 2022 study by Dr Zeman and colleagues described the potential ramifications of better understanding of the entire process: “Ultimately, investigation of the cellular and molecular mechanisms underlying ineffectual alveolar regeneration in ARDS and fibrosis may lead to novel therapies to promote physiological regeneration, thus accelerating restoration of barrier integrity, resolution of edema, liberation from the ventilator and survival in ARDS, and preventing fibrosis in fibroproliferative ARDS and [idiopathic pulmonary fibrosis].”

To put it more simply, “if you can seal the barrier, you can get the fluid out of the lungs, and you can get the patients off the ventilator, get out of the ICU, and go home,” Dr Zemans said.

Dr Zemans and Dr Martin have no disclosures.

Scientists are beginning to unravel the secrets of acute respiratory distress syndrome (ARDS), the devastating disorder that floods the lungs with fluid and has ushered countless millions to death after infection with pneumonia, sepsis, and COVID-19.

Two centuries after the lung damage caused by the disorder was first described in medicine, it’s now clear that ARDS is an autoimmune condition spurred by the body’s overactive defenses. There’s interest in disrupting “crosstalk” between cells, and rise of a new form of genetic analysis is allowing researchers to test their hypotheses more effectively than ever before. And, perhaps most importantly, recent findings reveal how stem cells in the epithelial lining of the lungs get stalled in an intermediate stage before regenerating into new cells. Reversing this process could trigger repair and recovery.

There’s still a ways to go before clinical trials can test therapies to turn things around at the epithelial level, acknowledged University of Michigan, Ann Arbor, professor of internal medicine Rachel L. Zemans, MD, in an interview. Still, “it’s a pretty exciting time,” said Dr Zemans, who manages a lab that explores how the lung epithelium responds to injury.
 

A lung disorder’s deep roots in human history

A British doctor first described the traits of ARDS in 1821, although this form of pulmonary edema had been described in “ancient writings,” according to a 2005 report by Gordon Bernard, MD, of Vanderbilt University Medical Center, Nashville, Tennessee. Sometimes called “double pneumonia,” ARDS was almost always fatal until the last few decades of the 20th century. “The advent of well-equipped ICUs, well-trained staff, and the availability of reliable positive pressure ventilators has allowed patients to be kept alive much longer and thus have the opportunity to heal the pulmonary injury and survive,” Dr Bernard wrote.

According to the Mayo Clinic, there are many causes of ARDS. Sepsis is the most common, and others include severe pneumonia, head/chest injuries, massive blood transfusions, pancreatitis, burns, and inhalation of harmful substances. Since 2020, ARDS has been a hallmark of COVID-19.

In an interview, University of Washington, Seattle, emeritus professor of medicine Thomas R. Martin MD, explained that ARDS occurs when the epithelium barrier in the lungs breaks down. Unlike the permeable endothelial barrier, the alveolar epithelium is “like a brick wall or a big dam, keeping red cells and plasma out of the airspace.”

In cases of pulmonary edema due to heart failure, fluid can back up into the lungs, said Dr Martin, who studies ARDS. However, pumps in the epithelium can clear that excess fluid pretty quickly because the epithelium remains in a normal state, he said. “Given enough time and enough medical support, people with heart failure and pulmonary edema can get better without lung injury.”

In ARDS, however, “the epithelium is damaged. Cells die in the alveolar wall, the scaffolding is exposed, and the fluid in the alveoli cannot be cleared out. You’ve got a disaster on your hands because all of the fluid and red blood cells and inflammatory products in the blood are going right into the airspace. The patient gets extremely short of breath because their oxygen level falls.”
 

 

 

COVID-19 virus finds a weak spot in the lungs

COVID-19 is “a classic example of an attack on the alveolar epithelium,” Dr Martin said.

By chance, the virus evolved to recognize receptors in the epithelium, allowing it to enter and propagate. “To make matters worse, defense mechanisms in the body attack those dying epithelial cells because the virus is visible on the surface cells. So lymphocytes from the immune system and macrophages attack the outer walls and cause further damage.”

Other scientists agree about this general picture of ARDS. “Studies of human lung tissue support the notion that failure of alveolar repair and regeneration mechanisms underlie the chronic lung dysfunction that can result from ARD,” wrote researchers from Cedars-Sinai Medical Center, Los Angeles, California, and Icahn School of Medicine at Mount Sinai, New York, NY, in a 2022 report.

According to Dr Martin, researchers and clinicians have discovered a pair of strategies to help vanquish COVID-19: Control viral entry through antiviral medication and dampen the body’s inflammatory response via steroids.

Still, “although we’ve learned lessons from COVID-19, we’re not good at all at promoting repair,” Dr Martin said. While new drugs have dramatically improved treatment for lung diseases such as cystic fibrosis, he said, “we don’t have good examples of new therapies that promote repair in ARDS.”
 

Looking for a way to turn the tide of fluid buildup

Dr Zemans and colleagues have uncovered a crucial obstacle to repair: the failure of stem cells to fully differentiate and become functional alveolar epithelial cells.

Researchers only began to understand a few years ago that the stem cells go through a transitional stage from type 2 to type 1, which make up 98% of cells in the epithelial surface, Dr Zemans said. In patients with ARDS who don’t get better quickly, “it looks like the cells get hung up in this intermediate state. They can’t finish that regeneration.”

As a 2022 study by Dr Zemans and colleagues put it, this process can lead to “ongoing barrier permeability, noncardiogenic pulmonary edema, and ventilator dependence, and mortality.” In fact, she said, “when we look at the lungs of people who died of ARDS, their cells were all in that intermediate stage.”

The discovery of the intermediate state only came about because of new technology called single-cell RNA sequencing, she said. “Now, these transitional cells are being found in other organs.”

Why do the epithelial cells get only part way through the regeneration process? It’s not entirely clear, Dr Zemans said, but researcher are intrigued by the idea that “cross-talk” between cells is playing a role.

“When the cells are in that stage, they also activate neighboring cells, including inflammatory cells, like macrophages, and fibroblasts,” she said. “And once those cells become activated, they become pathologic. What we think is that those cells then can talk back to the epithelial cells and prevent the epithelial cells from finishing that differentiation. It’s really hard to snap out of that positive feedback loop.”

This interaction probably evolved “for a good reason,” she said, “but it also became pathologic.” If the cells stay in the intermediate stage too long, she said, fibrosis develops. “They have scar tissue that never goes away. It takes a lot of work to expand the lungs when they’re so stiff when they should be stretchy like a rubber band. Scar tissue also gets in the way of the oxygen absorption, so some people have low oxygen levels.”
 

 

 

Future directions: Teaching cells to get “unstuck”

What’s next for research? One direction is exploring the variety of types of cells in the epithelium. Recent finding are revealing “new cell subpopulations that maintain alveolar homeostasis, communicate injury signals, and participate in normal and maladaptive repair. Emerging data illuminate the complexity of alveolar physiology and pathology to provide a more complete picture of how alveoli maintain health and respond to injurious stimuli,” write the researchers from Cedars-Sinai Medical Center and Icahn School of Medicine at Mount Sinai in their 2022 report.

Meanwhile, “we’re trying to look at the signaling pathways, the proteins or molecules, to understand the signals that tell a cell how to get unstuck,” Dr Zemans said. And researchers are exploring whether knocking out certain genes expressed by transitional cells in mice will lead to better outcomes, she said.

The 2022 study by Dr Zeman and colleagues described the potential ramifications of better understanding of the entire process: “Ultimately, investigation of the cellular and molecular mechanisms underlying ineffectual alveolar regeneration in ARDS and fibrosis may lead to novel therapies to promote physiological regeneration, thus accelerating restoration of barrier integrity, resolution of edema, liberation from the ventilator and survival in ARDS, and preventing fibrosis in fibroproliferative ARDS and [idiopathic pulmonary fibrosis].”

To put it more simply, “if you can seal the barrier, you can get the fluid out of the lungs, and you can get the patients off the ventilator, get out of the ICU, and go home,” Dr Zemans said.

Dr Zemans and Dr Martin have no disclosures.

Publications
Publications
Topics
Article Type
Sections
Disallow All Ads
Content Gating
No Gating (article Unlocked/Free)
Alternative CME
Disqus Comments
Default
Use ProPublica
Hide sidebar & use full width
render the right sidebar.
Conference Recap Checkbox
Not Conference Recap
Clinical Edge
Display the Slideshow in this Article
Medscape Article
Display survey writer
Reuters content
Disable Inline Native ads
WebMD Article

JAMA Internal Medicine Editor Recaps 2023’s High-Impact Research

Article Type
Changed
Tue, 01/02/2024 - 16:08

Harvard Medical School’s Sharon K. Inouye, MD, MPH, is editor in chief of JAMA Internal Medicine and a leading voice in American gerontology. We asked her to choose five of the influential journal’s most impactful studies from 2023 and highlight important take-home messages for internists and their colleagues.
 

Q: One of the studies you chose suggests that the antiviral nirmatrelvir (Paxlovid) can ward off long COVID. Could you recap the findings?

A: Researchers followed a group of more than 280,000 Department of Veterans Affairs patients who were seen in 2022, had a positive COVID test, and had at least one risk factor for severe COVID. They focused on those who survived to 30 days after their COVID infection and compared those who received the drug within the first 5 days of a positive test with an equivalent control group.

They found that 13 long COVID symptoms were all significantly less common (relative risk = 0.74) in those who received nirmatrelvir. This was true no matter whether they’d ever had a COVID vaccination.
 

Q: How should this research affect clinical practice?

A: You can’t generalize from this to everyone because, of course, not everyone was included in this study. But it is highly suggestive that this drug is very effective for preventing long COVID.

Nirmatrelvir was touted as being able to shorten duration of illness and prevent hospitalization. But if you were low risk or you were already well into your COVID course, it wasn’t like rush, rush, rush to the doctor to get it.

This changes that equation because we know long COVID is such a huge issue. The vast majority of doctors who work with COVID patients and know this are now being more aggressive about prescribing it.
 

Q: What about patients whom the CDC considers to be at less risk — people with up-to-date vaccinations who are under 50 with mild-to-moderate COVID and no higher-risk medical conditions? Should they take nirmatrelvir?

A: The evidence is not 100% in yet. A study like this one needs to be repeated and include younger people without any risk factors to see if we see the same thing. So it’s a personal choice, and a personal calculus needs to be done. A lot of people are making that choice [to take the drug], and it can be a rational decision.

Q: You also chose a study that links high thyroid hormone levels to higher rates of dementia. What did it reveal?

A: This study looks at patients who had thyrotoxicosis — a thyroid level that’s too high — from hormone produced endogenously, and exogenously. Researchers tracked almost 66,000 patients aged 65 and older and found that thyrotoxicosis from all causes, whether it was endogenous or exogenous, was linked to an increased risk of dementia in a dose-response relationship (adjusted hazard ratio = 1.39).

Q: Is there a clinical take-home message here?

A: When we start patients on thyroid medication, they don’t always get reassessed on a regular basis. Given this finding, a TSH [thyroid-stimulating hormone] level is indicated during the annual wellness check that patients on Medicare can get every year.

 

 

Q: Is TSH measured as part of routine blood tests?

A: No it’s not. It has to be ordered. I think that’s why we’re seeing this problem to begin with — because it’s not something we all have awareness about. I wasn’t aware myself that mildly high levels of thyroid could increase the risk of cognitive impairment. Certainly, I’m going to be much more aware in my practice.

Q: You also picked a study about silicosis in workers who are exposed to dust when they make engineered stone countertops, also known as quartz countertops. What were the findings?

A: Silicosis is a very serious lung condition that develops from exposure to crystalline silica. Essentially, sand gets inhaled into the lungs. Workers can be exposed when they’re making engineered stone countertops, the most popular countertops now in the United States.

This study is based on statewide surveys from 2019 to 2022 that the California Department of Public Health does routinely. They gathered cases of silicosis and found 52 — all men with an average age of 45. All but one were Latino immigrants, and most either had no insurance or very poor insurance.
 

Q: The study found that “diagnosis was delayed in 58%, with 38% presenting with advanced disease (progressive massive fibrosis), and 19% died.” What does that tell you?

A: It’s a very serious condition. Once it gets to the advanced stage, it will just continue to progress, and the person will die. That’s why it’s so important to know that it’s absolutely preventable.

Q: Is there a message here for internists?

A: If you treat a lot of immigrants or work in an area where there are a lot of industrial workers, you’re going to want to have a very high suspicion about it. If you see an atypical pattern on the chest x-ray or via diffusion scoring, have a low threshold for getting a pulmonary function test.

Doctors need to be aware and diagnose this very quickly. When patients present, you can pull them out of that work environment or put mitigation systems into place.
 

Q: California regulators were expected to put emergency rules into place in late December to protect workers. Did this study play a role in focusing attention on the problem?

A: This article, along with a commentary and podcast that we put out, really helped with advocacy to improve health and safety for workers at stone-cutting and fabrication shops.

Q: You were impressed by another study about airborne dangers, this one linking air pollution to dementia. What did researchers discover?

A: [This analysis] of more than 27,000 people in the Health and Retirement Study, a respected and rich database, found that exposure to air pollution was associated with greater rates of dementia — an increase of about 8% a year. Exposure to agricultural emissions and wildfire smoke were most robustly associated with a greater risk of dementia.

Q: How are these findings important, especially in light of the unhealthy air spawned by recent wildfires in the United States and Canada?

A: Studies like this will make it even more compelling that we are better prepared for air quality issues.

I grew up in Los Angeles, where smog and pollution were very big issues. I was constantly hearing about various mitigation strategies that were going into place. But after I moved to the East Coast, I almost never heard about prevention.

Now, I’m hoping we can keep this topic in the national conversation.
 

Q: You also highlighted a systematic review of the use of restraints in the emergency department. Why did you choose this research?

A: At JAMA Internal Medicine, we’re really focused on ways we can address health disparities and raise awareness of potential unconscious bias.

This review looked at 10 studies that included more than 2.5 million patient encounters, including 24,000 incidents of physical restraint use. They found that the overall rate of use of restraints was low at below 1%.

But when they are used, Black patients were 1.3 times more likely to be restrained than White patients.
 

Q: What’s the message here?

A: This is an important start to recognizing these differences and then changing our behavior. Perhaps restraints don’t need to be used as often in light of evidence, for example, of increased rates of misdiagnosis of psychosis in the Black population.

Q: How should physicians change their approach to restraints?

A: Restraints are not to be used to control disruption — wild behavior or verbal outbursts. They’re for when someone is a danger to themselves or others.

Dr. Inouye has no conflicts of interest.

Publications
Topics
Sections

Harvard Medical School’s Sharon K. Inouye, MD, MPH, is editor in chief of JAMA Internal Medicine and a leading voice in American gerontology. We asked her to choose five of the influential journal’s most impactful studies from 2023 and highlight important take-home messages for internists and their colleagues.
 

Q: One of the studies you chose suggests that the antiviral nirmatrelvir (Paxlovid) can ward off long COVID. Could you recap the findings?

A: Researchers followed a group of more than 280,000 Department of Veterans Affairs patients who were seen in 2022, had a positive COVID test, and had at least one risk factor for severe COVID. They focused on those who survived to 30 days after their COVID infection and compared those who received the drug within the first 5 days of a positive test with an equivalent control group.

They found that 13 long COVID symptoms were all significantly less common (relative risk = 0.74) in those who received nirmatrelvir. This was true no matter whether they’d ever had a COVID vaccination.
 

Q: How should this research affect clinical practice?

A: You can’t generalize from this to everyone because, of course, not everyone was included in this study. But it is highly suggestive that this drug is very effective for preventing long COVID.

Nirmatrelvir was touted as being able to shorten duration of illness and prevent hospitalization. But if you were low risk or you were already well into your COVID course, it wasn’t like rush, rush, rush to the doctor to get it.

This changes that equation because we know long COVID is such a huge issue. The vast majority of doctors who work with COVID patients and know this are now being more aggressive about prescribing it.
 

Q: What about patients whom the CDC considers to be at less risk — people with up-to-date vaccinations who are under 50 with mild-to-moderate COVID and no higher-risk medical conditions? Should they take nirmatrelvir?

A: The evidence is not 100% in yet. A study like this one needs to be repeated and include younger people without any risk factors to see if we see the same thing. So it’s a personal choice, and a personal calculus needs to be done. A lot of people are making that choice [to take the drug], and it can be a rational decision.

Q: You also chose a study that links high thyroid hormone levels to higher rates of dementia. What did it reveal?

A: This study looks at patients who had thyrotoxicosis — a thyroid level that’s too high — from hormone produced endogenously, and exogenously. Researchers tracked almost 66,000 patients aged 65 and older and found that thyrotoxicosis from all causes, whether it was endogenous or exogenous, was linked to an increased risk of dementia in a dose-response relationship (adjusted hazard ratio = 1.39).

Q: Is there a clinical take-home message here?

A: When we start patients on thyroid medication, they don’t always get reassessed on a regular basis. Given this finding, a TSH [thyroid-stimulating hormone] level is indicated during the annual wellness check that patients on Medicare can get every year.

 

 

Q: Is TSH measured as part of routine blood tests?

A: No it’s not. It has to be ordered. I think that’s why we’re seeing this problem to begin with — because it’s not something we all have awareness about. I wasn’t aware myself that mildly high levels of thyroid could increase the risk of cognitive impairment. Certainly, I’m going to be much more aware in my practice.

Q: You also picked a study about silicosis in workers who are exposed to dust when they make engineered stone countertops, also known as quartz countertops. What were the findings?

A: Silicosis is a very serious lung condition that develops from exposure to crystalline silica. Essentially, sand gets inhaled into the lungs. Workers can be exposed when they’re making engineered stone countertops, the most popular countertops now in the United States.

This study is based on statewide surveys from 2019 to 2022 that the California Department of Public Health does routinely. They gathered cases of silicosis and found 52 — all men with an average age of 45. All but one were Latino immigrants, and most either had no insurance or very poor insurance.
 

Q: The study found that “diagnosis was delayed in 58%, with 38% presenting with advanced disease (progressive massive fibrosis), and 19% died.” What does that tell you?

A: It’s a very serious condition. Once it gets to the advanced stage, it will just continue to progress, and the person will die. That’s why it’s so important to know that it’s absolutely preventable.

Q: Is there a message here for internists?

A: If you treat a lot of immigrants or work in an area where there are a lot of industrial workers, you’re going to want to have a very high suspicion about it. If you see an atypical pattern on the chest x-ray or via diffusion scoring, have a low threshold for getting a pulmonary function test.

Doctors need to be aware and diagnose this very quickly. When patients present, you can pull them out of that work environment or put mitigation systems into place.
 

Q: California regulators were expected to put emergency rules into place in late December to protect workers. Did this study play a role in focusing attention on the problem?

A: This article, along with a commentary and podcast that we put out, really helped with advocacy to improve health and safety for workers at stone-cutting and fabrication shops.

Q: You were impressed by another study about airborne dangers, this one linking air pollution to dementia. What did researchers discover?

A: [This analysis] of more than 27,000 people in the Health and Retirement Study, a respected and rich database, found that exposure to air pollution was associated with greater rates of dementia — an increase of about 8% a year. Exposure to agricultural emissions and wildfire smoke were most robustly associated with a greater risk of dementia.

Q: How are these findings important, especially in light of the unhealthy air spawned by recent wildfires in the United States and Canada?

A: Studies like this will make it even more compelling that we are better prepared for air quality issues.

I grew up in Los Angeles, where smog and pollution were very big issues. I was constantly hearing about various mitigation strategies that were going into place. But after I moved to the East Coast, I almost never heard about prevention.

Now, I’m hoping we can keep this topic in the national conversation.
 

Q: You also highlighted a systematic review of the use of restraints in the emergency department. Why did you choose this research?

A: At JAMA Internal Medicine, we’re really focused on ways we can address health disparities and raise awareness of potential unconscious bias.

This review looked at 10 studies that included more than 2.5 million patient encounters, including 24,000 incidents of physical restraint use. They found that the overall rate of use of restraints was low at below 1%.

But when they are used, Black patients were 1.3 times more likely to be restrained than White patients.
 

Q: What’s the message here?

A: This is an important start to recognizing these differences and then changing our behavior. Perhaps restraints don’t need to be used as often in light of evidence, for example, of increased rates of misdiagnosis of psychosis in the Black population.

Q: How should physicians change their approach to restraints?

A: Restraints are not to be used to control disruption — wild behavior or verbal outbursts. They’re for when someone is a danger to themselves or others.

Dr. Inouye has no conflicts of interest.

Harvard Medical School’s Sharon K. Inouye, MD, MPH, is editor in chief of JAMA Internal Medicine and a leading voice in American gerontology. We asked her to choose five of the influential journal’s most impactful studies from 2023 and highlight important take-home messages for internists and their colleagues.
 

Q: One of the studies you chose suggests that the antiviral nirmatrelvir (Paxlovid) can ward off long COVID. Could you recap the findings?

A: Researchers followed a group of more than 280,000 Department of Veterans Affairs patients who were seen in 2022, had a positive COVID test, and had at least one risk factor for severe COVID. They focused on those who survived to 30 days after their COVID infection and compared those who received the drug within the first 5 days of a positive test with an equivalent control group.

They found that 13 long COVID symptoms were all significantly less common (relative risk = 0.74) in those who received nirmatrelvir. This was true no matter whether they’d ever had a COVID vaccination.
 

Q: How should this research affect clinical practice?

A: You can’t generalize from this to everyone because, of course, not everyone was included in this study. But it is highly suggestive that this drug is very effective for preventing long COVID.

Nirmatrelvir was touted as being able to shorten duration of illness and prevent hospitalization. But if you were low risk or you were already well into your COVID course, it wasn’t like rush, rush, rush to the doctor to get it.

This changes that equation because we know long COVID is such a huge issue. The vast majority of doctors who work with COVID patients and know this are now being more aggressive about prescribing it.
 

Q: What about patients whom the CDC considers to be at less risk — people with up-to-date vaccinations who are under 50 with mild-to-moderate COVID and no higher-risk medical conditions? Should they take nirmatrelvir?

A: The evidence is not 100% in yet. A study like this one needs to be repeated and include younger people without any risk factors to see if we see the same thing. So it’s a personal choice, and a personal calculus needs to be done. A lot of people are making that choice [to take the drug], and it can be a rational decision.

Q: You also chose a study that links high thyroid hormone levels to higher rates of dementia. What did it reveal?

A: This study looks at patients who had thyrotoxicosis — a thyroid level that’s too high — from hormone produced endogenously, and exogenously. Researchers tracked almost 66,000 patients aged 65 and older and found that thyrotoxicosis from all causes, whether it was endogenous or exogenous, was linked to an increased risk of dementia in a dose-response relationship (adjusted hazard ratio = 1.39).

Q: Is there a clinical take-home message here?

A: When we start patients on thyroid medication, they don’t always get reassessed on a regular basis. Given this finding, a TSH [thyroid-stimulating hormone] level is indicated during the annual wellness check that patients on Medicare can get every year.

 

 

Q: Is TSH measured as part of routine blood tests?

A: No it’s not. It has to be ordered. I think that’s why we’re seeing this problem to begin with — because it’s not something we all have awareness about. I wasn’t aware myself that mildly high levels of thyroid could increase the risk of cognitive impairment. Certainly, I’m going to be much more aware in my practice.

Q: You also picked a study about silicosis in workers who are exposed to dust when they make engineered stone countertops, also known as quartz countertops. What were the findings?

A: Silicosis is a very serious lung condition that develops from exposure to crystalline silica. Essentially, sand gets inhaled into the lungs. Workers can be exposed when they’re making engineered stone countertops, the most popular countertops now in the United States.

This study is based on statewide surveys from 2019 to 2022 that the California Department of Public Health does routinely. They gathered cases of silicosis and found 52 — all men with an average age of 45. All but one were Latino immigrants, and most either had no insurance or very poor insurance.
 

Q: The study found that “diagnosis was delayed in 58%, with 38% presenting with advanced disease (progressive massive fibrosis), and 19% died.” What does that tell you?

A: It’s a very serious condition. Once it gets to the advanced stage, it will just continue to progress, and the person will die. That’s why it’s so important to know that it’s absolutely preventable.

Q: Is there a message here for internists?

A: If you treat a lot of immigrants or work in an area where there are a lot of industrial workers, you’re going to want to have a very high suspicion about it. If you see an atypical pattern on the chest x-ray or via diffusion scoring, have a low threshold for getting a pulmonary function test.

Doctors need to be aware and diagnose this very quickly. When patients present, you can pull them out of that work environment or put mitigation systems into place.
 

Q: California regulators were expected to put emergency rules into place in late December to protect workers. Did this study play a role in focusing attention on the problem?

A: This article, along with a commentary and podcast that we put out, really helped with advocacy to improve health and safety for workers at stone-cutting and fabrication shops.

Q: You were impressed by another study about airborne dangers, this one linking air pollution to dementia. What did researchers discover?

A: [This analysis] of more than 27,000 people in the Health and Retirement Study, a respected and rich database, found that exposure to air pollution was associated with greater rates of dementia — an increase of about 8% a year. Exposure to agricultural emissions and wildfire smoke were most robustly associated with a greater risk of dementia.

Q: How are these findings important, especially in light of the unhealthy air spawned by recent wildfires in the United States and Canada?

A: Studies like this will make it even more compelling that we are better prepared for air quality issues.

I grew up in Los Angeles, where smog and pollution were very big issues. I was constantly hearing about various mitigation strategies that were going into place. But after I moved to the East Coast, I almost never heard about prevention.

Now, I’m hoping we can keep this topic in the national conversation.
 

Q: You also highlighted a systematic review of the use of restraints in the emergency department. Why did you choose this research?

A: At JAMA Internal Medicine, we’re really focused on ways we can address health disparities and raise awareness of potential unconscious bias.

This review looked at 10 studies that included more than 2.5 million patient encounters, including 24,000 incidents of physical restraint use. They found that the overall rate of use of restraints was low at below 1%.

But when they are used, Black patients were 1.3 times more likely to be restrained than White patients.
 

Q: What’s the message here?

A: This is an important start to recognizing these differences and then changing our behavior. Perhaps restraints don’t need to be used as often in light of evidence, for example, of increased rates of misdiagnosis of psychosis in the Black population.

Q: How should physicians change their approach to restraints?

A: Restraints are not to be used to control disruption — wild behavior or verbal outbursts. They’re for when someone is a danger to themselves or others.

Dr. Inouye has no conflicts of interest.

Publications
Publications
Topics
Article Type
Sections
Disallow All Ads
Content Gating
No Gating (article Unlocked/Free)
Alternative CME
Disqus Comments
Default
Use ProPublica
Hide sidebar & use full width
render the right sidebar.
Conference Recap Checkbox
Not Conference Recap
Clinical Edge
Display the Slideshow in this Article
Medscape Article
Display survey writer
Reuters content
Disable Inline Native ads
WebMD Article

Olympus provides recall correction for bronchoscopes to reduce fire risk

Article Type
Changed
Thu, 12/28/2023 - 13:07

A series of Olympus bronchofiberscopes and bronchovideoscopes have been recalled by the manufacturer because of a risk for burns and fire, according to a statement from the US Food and Drug Administration (FDA). 

However, “this recall is a correction, not a product removal,” according to the FDA. Clinicians do not need to cease using these devices, but they must be mindful of the risks and take the precautions outlined by Olympus.

“While health care providers may choose to continue using the Olympus bronchofiberscopes and bronchovideoscopes, to maximize patient safety and mitigate any potential risk to patient health, the FDA and Olympus advise users not to perform high-frequency cauterization while supplying oxygen, and carefully follow the warnings provided in the Olympus operators manual and highlighted in its October 12, 2023, letter to customers,” an FDA spokesperson said.

The recall affects Olympus bronchofiberscopes and bronchovideoscopes distributed between January 1, 2001, and September 11, 2023. According to the FDA statement, use of these devices may cause serious adverse events to patients and to clinicians. Patients treated with these devices could experience critical burns in the airways or lungs, airway bleeding, breathing difficulty, apnea, loss of consciousness, or death. Healthcare workers using the devices also may be affected in the event of combustion. 

On October 12, 2023, Olympus sent an Urgent Medical Device Corrective Action letter. This letter outlined the risks associated with the devices as follows: 

“There is a risk of endobronchial combustion if high-frequency cauterization is performed while supplying oxygen [and/or] the electrode section of the electrosurgical accessory is too close to the distal end of the endoscope.” 

To mitigate this risk, Olympus reminds clinicians to heed the warnings found in the device operations manuals, notably these three: 

  • Do not perform high-frequency cauterization while supplying oxygen.
  • Confirm that the electrode section of the electrosurgical device used with the endoscope is at a safe distance from the distal end of the endoscope.
  • Only use the Olympus bronchoscopes with compatible high-frequency therapy equipment as described in the operations manual.

The letter also asks facilities that have purchased any of the affected bronchoscopes to ensure that all personnel are “completely knowledgeable and thoroughly aware” of the warnings stated in the operations manual, and it states that users may continue to use the devices according to the current instructions and with attention to the warnings.
 

Olympus Explains

“Olympus Corporation initiated this Field Corrective Action (FCA) to address complaints of endobronchial combustion occurring when high-frequency-compatible bronchoscopes are used during therapeutic procedures in combination with high-frequency therapy equipment,” a spokeswoman for Olympus said in an interview. 

“This corrective action was taken following a thorough assessment of adverse event complaints involving serious patient injury; Olympus takes these complaints very seriously. Patient safety is our top priority,” the spokeswoman said. “The customer notification is intended to remind users of existing warnings not to use oxygen while performing high-frequency cauterization and appropriate distance while using high-frequency therapy equipment.” 

The products are not being removed, and no labeling changes are being made at this time, she said. 

The bottom line for clinicians: “Users can continue to use Olympus bronchoscopes according to the instructions provided in the operation manual and the customer letter,” the Olympus spokeswoman told this news organization. “This is not a removal action. There are no changes to the existing operation manual regarding compatibility of bronchoscopes with high-frequency therapy equipment,” she said.

“In terms of actions going forward, in addition to the communication provided through this Field Corrective Action, which is intended to remind users of recommendations on oxygen use and clarify the appropriate distance while using high-frequency therapy equipment, the root cause and potential contributing factors are currently under investigation through a formal CAPA (Corrective Action Preventative Action) process. Olympus will take any appropriate enhancement action based on investigation results,” according to the Olympus spokeswoman.

In 2016, this news organization reported that Olympus made medical headlines by recalling its TJF-Q180V duodenoscope in the wake of Congressional investigations after the product was linked to spreading bacterial infections because of design flaws. 

United States customers can contact Olympus by phone at 1-800-848-9024 (option 1) with questions about the recall, and healthcare professionals and consumers may report adverse reactions or quality problems associated with the devices to MedWatch: The FDA Safety Information and Adverse Event Reporting Program via an online form, regular mail, or fax.

A version of this article first appeared on Medscape.com.

Publications
Topics
Sections

A series of Olympus bronchofiberscopes and bronchovideoscopes have been recalled by the manufacturer because of a risk for burns and fire, according to a statement from the US Food and Drug Administration (FDA). 

However, “this recall is a correction, not a product removal,” according to the FDA. Clinicians do not need to cease using these devices, but they must be mindful of the risks and take the precautions outlined by Olympus.

“While health care providers may choose to continue using the Olympus bronchofiberscopes and bronchovideoscopes, to maximize patient safety and mitigate any potential risk to patient health, the FDA and Olympus advise users not to perform high-frequency cauterization while supplying oxygen, and carefully follow the warnings provided in the Olympus operators manual and highlighted in its October 12, 2023, letter to customers,” an FDA spokesperson said.

The recall affects Olympus bronchofiberscopes and bronchovideoscopes distributed between January 1, 2001, and September 11, 2023. According to the FDA statement, use of these devices may cause serious adverse events to patients and to clinicians. Patients treated with these devices could experience critical burns in the airways or lungs, airway bleeding, breathing difficulty, apnea, loss of consciousness, or death. Healthcare workers using the devices also may be affected in the event of combustion. 

On October 12, 2023, Olympus sent an Urgent Medical Device Corrective Action letter. This letter outlined the risks associated with the devices as follows: 

“There is a risk of endobronchial combustion if high-frequency cauterization is performed while supplying oxygen [and/or] the electrode section of the electrosurgical accessory is too close to the distal end of the endoscope.” 

To mitigate this risk, Olympus reminds clinicians to heed the warnings found in the device operations manuals, notably these three: 

  • Do not perform high-frequency cauterization while supplying oxygen.
  • Confirm that the electrode section of the electrosurgical device used with the endoscope is at a safe distance from the distal end of the endoscope.
  • Only use the Olympus bronchoscopes with compatible high-frequency therapy equipment as described in the operations manual.

The letter also asks facilities that have purchased any of the affected bronchoscopes to ensure that all personnel are “completely knowledgeable and thoroughly aware” of the warnings stated in the operations manual, and it states that users may continue to use the devices according to the current instructions and with attention to the warnings.
 

Olympus Explains

“Olympus Corporation initiated this Field Corrective Action (FCA) to address complaints of endobronchial combustion occurring when high-frequency-compatible bronchoscopes are used during therapeutic procedures in combination with high-frequency therapy equipment,” a spokeswoman for Olympus said in an interview. 

“This corrective action was taken following a thorough assessment of adverse event complaints involving serious patient injury; Olympus takes these complaints very seriously. Patient safety is our top priority,” the spokeswoman said. “The customer notification is intended to remind users of existing warnings not to use oxygen while performing high-frequency cauterization and appropriate distance while using high-frequency therapy equipment.” 

The products are not being removed, and no labeling changes are being made at this time, she said. 

The bottom line for clinicians: “Users can continue to use Olympus bronchoscopes according to the instructions provided in the operation manual and the customer letter,” the Olympus spokeswoman told this news organization. “This is not a removal action. There are no changes to the existing operation manual regarding compatibility of bronchoscopes with high-frequency therapy equipment,” she said.

“In terms of actions going forward, in addition to the communication provided through this Field Corrective Action, which is intended to remind users of recommendations on oxygen use and clarify the appropriate distance while using high-frequency therapy equipment, the root cause and potential contributing factors are currently under investigation through a formal CAPA (Corrective Action Preventative Action) process. Olympus will take any appropriate enhancement action based on investigation results,” according to the Olympus spokeswoman.

In 2016, this news organization reported that Olympus made medical headlines by recalling its TJF-Q180V duodenoscope in the wake of Congressional investigations after the product was linked to spreading bacterial infections because of design flaws. 

United States customers can contact Olympus by phone at 1-800-848-9024 (option 1) with questions about the recall, and healthcare professionals and consumers may report adverse reactions or quality problems associated with the devices to MedWatch: The FDA Safety Information and Adverse Event Reporting Program via an online form, regular mail, or fax.

A version of this article first appeared on Medscape.com.

A series of Olympus bronchofiberscopes and bronchovideoscopes have been recalled by the manufacturer because of a risk for burns and fire, according to a statement from the US Food and Drug Administration (FDA). 

However, “this recall is a correction, not a product removal,” according to the FDA. Clinicians do not need to cease using these devices, but they must be mindful of the risks and take the precautions outlined by Olympus.

“While health care providers may choose to continue using the Olympus bronchofiberscopes and bronchovideoscopes, to maximize patient safety and mitigate any potential risk to patient health, the FDA and Olympus advise users not to perform high-frequency cauterization while supplying oxygen, and carefully follow the warnings provided in the Olympus operators manual and highlighted in its October 12, 2023, letter to customers,” an FDA spokesperson said.

The recall affects Olympus bronchofiberscopes and bronchovideoscopes distributed between January 1, 2001, and September 11, 2023. According to the FDA statement, use of these devices may cause serious adverse events to patients and to clinicians. Patients treated with these devices could experience critical burns in the airways or lungs, airway bleeding, breathing difficulty, apnea, loss of consciousness, or death. Healthcare workers using the devices also may be affected in the event of combustion. 

On October 12, 2023, Olympus sent an Urgent Medical Device Corrective Action letter. This letter outlined the risks associated with the devices as follows: 

“There is a risk of endobronchial combustion if high-frequency cauterization is performed while supplying oxygen [and/or] the electrode section of the electrosurgical accessory is too close to the distal end of the endoscope.” 

To mitigate this risk, Olympus reminds clinicians to heed the warnings found in the device operations manuals, notably these three: 

  • Do not perform high-frequency cauterization while supplying oxygen.
  • Confirm that the electrode section of the electrosurgical device used with the endoscope is at a safe distance from the distal end of the endoscope.
  • Only use the Olympus bronchoscopes with compatible high-frequency therapy equipment as described in the operations manual.

The letter also asks facilities that have purchased any of the affected bronchoscopes to ensure that all personnel are “completely knowledgeable and thoroughly aware” of the warnings stated in the operations manual, and it states that users may continue to use the devices according to the current instructions and with attention to the warnings.
 

Olympus Explains

“Olympus Corporation initiated this Field Corrective Action (FCA) to address complaints of endobronchial combustion occurring when high-frequency-compatible bronchoscopes are used during therapeutic procedures in combination with high-frequency therapy equipment,” a spokeswoman for Olympus said in an interview. 

“This corrective action was taken following a thorough assessment of adverse event complaints involving serious patient injury; Olympus takes these complaints very seriously. Patient safety is our top priority,” the spokeswoman said. “The customer notification is intended to remind users of existing warnings not to use oxygen while performing high-frequency cauterization and appropriate distance while using high-frequency therapy equipment.” 

The products are not being removed, and no labeling changes are being made at this time, she said. 

The bottom line for clinicians: “Users can continue to use Olympus bronchoscopes according to the instructions provided in the operation manual and the customer letter,” the Olympus spokeswoman told this news organization. “This is not a removal action. There are no changes to the existing operation manual regarding compatibility of bronchoscopes with high-frequency therapy equipment,” she said.

“In terms of actions going forward, in addition to the communication provided through this Field Corrective Action, which is intended to remind users of recommendations on oxygen use and clarify the appropriate distance while using high-frequency therapy equipment, the root cause and potential contributing factors are currently under investigation through a formal CAPA (Corrective Action Preventative Action) process. Olympus will take any appropriate enhancement action based on investigation results,” according to the Olympus spokeswoman.

In 2016, this news organization reported that Olympus made medical headlines by recalling its TJF-Q180V duodenoscope in the wake of Congressional investigations after the product was linked to spreading bacterial infections because of design flaws. 

United States customers can contact Olympus by phone at 1-800-848-9024 (option 1) with questions about the recall, and healthcare professionals and consumers may report adverse reactions or quality problems associated with the devices to MedWatch: The FDA Safety Information and Adverse Event Reporting Program via an online form, regular mail, or fax.

A version of this article first appeared on Medscape.com.

Publications
Publications
Topics
Article Type
Sections
Disallow All Ads
Content Gating
No Gating (article Unlocked/Free)
Alternative CME
Disqus Comments
Default
Use ProPublica
Hide sidebar & use full width
render the right sidebar.
Conference Recap Checkbox
Not Conference Recap
Clinical Edge
Display the Slideshow in this Article
Medscape Article
Display survey writer
Reuters content
Disable Inline Native ads
WebMD Article

Death Risk Takes Decades to Revert to Normal in Ex-Smokers

Article Type
Changed
Wed, 12/27/2023 - 12:06

For smokers, deaths with a cardiovascular or cancer-related cause, or ones that can be attributed to a respiratory disease such as chronic obstructive pulmonary disease, are significantly more common than for nonsmokers. It is widely recognized that stopping smoking leads to a reduction in mortality risk. To make reliable statements on the timeline of this reduction, researchers analyzed interview data and death rates from 438,015 adult US citizens from 1997 to the end of 2019.

The analyses show that it takes 30 years for the mortality risk of ex-smokers to resemble that of people who never regularly smoked. Blake Thomson, PhD, and Fahrad Islami, MD, PhD, both members of the Department of Surveillance and Health Equity Science of the American Cancer Society in Atlanta, Georgia, published their results as a research letter in JAMA Internal Medicine.
 

After Smoking Cessation

Overall, 11,860 cardiovascular, 10,935 cancer-related, and 2,060 respiratory-related deaths were considered from over 5 million patient years. Taken from these figures, the mortality risks of continuous smokers were 2.3 times (cardiovascular), 3.4 times (cancer-related), and 13.3 times (respiratory-related) higher than those of continuous nonsmokers.

Within 10 years of stopping smoking, the following occurred:

  • The cardiovascular mortality risk fell by 1.47 times, compared with nonsmokers (by 36% compared with smokers).
  • The cancer-related mortality risk fell by 2.13 times, compared with nonsmokers (by 47% compared with smokers).
  • The respiratory-related mortality risk fell by 6.35 times, compared with nonsmokers (by 43% compared with smokers).

In the second decade after stopping smoking, the risk dropped even further. The researchers observed the following trends:

  • The cardiovascular mortality risk fell by 1.26 times.
  • The cancer-related mortality risk fell by 1.59 times.
  • The respiratory-related mortality risk fell by 3.63 times — each time compared with nonsmokers.

During the third decade after stopping smoking, the risk continued to decrease. The trends were as follows:

  • The cardiovascular mortality risk fell by 1.07 times.
  • The cancer-related mortality risk fell by 1.34 times.
  • The respiratory-related mortality risk fell by 2.34 times, compared with nonsmokers.

30 Years Later

Only after more than 30 years of not smoking was the cardiovascular-related mortality risk 0.96 and, therefore, no longer significant. Compared with nonsmokers, the cancer-related mortality risk was 1.16, and the respiratory-related mortality risk was 1.31.

Therefore, former smokers can reduce their cardiovascular mortality risk by 100%, the cancer-related by 93%, and the respiratory-related mortality risk by 97%.

The result reinforces earlier analyses on the reduction in mortality risks by stopping smoking, with fewer participants. Smokers, therefore, benefit more the longer that they can refrain from using tobacco. “The earlier in life that smoking is given up, the better,” the authors wrote. But even in the first 10 years, the mortality risks examined decreased by a statistically significant 36% (cardiovascular) to 47% (cancer-related).
 

An Underestimation?

One disadvantage of the study is that the participants’ data were collected using personal questionnaires. For this reason, participants may have reported their tobacco consumption as being lower than it was, particularly because these questionnaires are often answered in hindsight, the authors pointed out.

In addition, some of the participants who reported stopping smoking completely may have only reduced their consumption. However, both circumstances would cause the results of the analysis to be even clearer, compared with reality, and therefore better.

This article was translated from the Medscape German edition.

A version of this article appeared on Medscape.com.

Publications
Topics
Sections

For smokers, deaths with a cardiovascular or cancer-related cause, or ones that can be attributed to a respiratory disease such as chronic obstructive pulmonary disease, are significantly more common than for nonsmokers. It is widely recognized that stopping smoking leads to a reduction in mortality risk. To make reliable statements on the timeline of this reduction, researchers analyzed interview data and death rates from 438,015 adult US citizens from 1997 to the end of 2019.

The analyses show that it takes 30 years for the mortality risk of ex-smokers to resemble that of people who never regularly smoked. Blake Thomson, PhD, and Fahrad Islami, MD, PhD, both members of the Department of Surveillance and Health Equity Science of the American Cancer Society in Atlanta, Georgia, published their results as a research letter in JAMA Internal Medicine.
 

After Smoking Cessation

Overall, 11,860 cardiovascular, 10,935 cancer-related, and 2,060 respiratory-related deaths were considered from over 5 million patient years. Taken from these figures, the mortality risks of continuous smokers were 2.3 times (cardiovascular), 3.4 times (cancer-related), and 13.3 times (respiratory-related) higher than those of continuous nonsmokers.

Within 10 years of stopping smoking, the following occurred:

  • The cardiovascular mortality risk fell by 1.47 times, compared with nonsmokers (by 36% compared with smokers).
  • The cancer-related mortality risk fell by 2.13 times, compared with nonsmokers (by 47% compared with smokers).
  • The respiratory-related mortality risk fell by 6.35 times, compared with nonsmokers (by 43% compared with smokers).

In the second decade after stopping smoking, the risk dropped even further. The researchers observed the following trends:

  • The cardiovascular mortality risk fell by 1.26 times.
  • The cancer-related mortality risk fell by 1.59 times.
  • The respiratory-related mortality risk fell by 3.63 times — each time compared with nonsmokers.

During the third decade after stopping smoking, the risk continued to decrease. The trends were as follows:

  • The cardiovascular mortality risk fell by 1.07 times.
  • The cancer-related mortality risk fell by 1.34 times.
  • The respiratory-related mortality risk fell by 2.34 times, compared with nonsmokers.

30 Years Later

Only after more than 30 years of not smoking was the cardiovascular-related mortality risk 0.96 and, therefore, no longer significant. Compared with nonsmokers, the cancer-related mortality risk was 1.16, and the respiratory-related mortality risk was 1.31.

Therefore, former smokers can reduce their cardiovascular mortality risk by 100%, the cancer-related by 93%, and the respiratory-related mortality risk by 97%.

The result reinforces earlier analyses on the reduction in mortality risks by stopping smoking, with fewer participants. Smokers, therefore, benefit more the longer that they can refrain from using tobacco. “The earlier in life that smoking is given up, the better,” the authors wrote. But even in the first 10 years, the mortality risks examined decreased by a statistically significant 36% (cardiovascular) to 47% (cancer-related).
 

An Underestimation?

One disadvantage of the study is that the participants’ data were collected using personal questionnaires. For this reason, participants may have reported their tobacco consumption as being lower than it was, particularly because these questionnaires are often answered in hindsight, the authors pointed out.

In addition, some of the participants who reported stopping smoking completely may have only reduced their consumption. However, both circumstances would cause the results of the analysis to be even clearer, compared with reality, and therefore better.

This article was translated from the Medscape German edition.

A version of this article appeared on Medscape.com.

For smokers, deaths with a cardiovascular or cancer-related cause, or ones that can be attributed to a respiratory disease such as chronic obstructive pulmonary disease, are significantly more common than for nonsmokers. It is widely recognized that stopping smoking leads to a reduction in mortality risk. To make reliable statements on the timeline of this reduction, researchers analyzed interview data and death rates from 438,015 adult US citizens from 1997 to the end of 2019.

The analyses show that it takes 30 years for the mortality risk of ex-smokers to resemble that of people who never regularly smoked. Blake Thomson, PhD, and Fahrad Islami, MD, PhD, both members of the Department of Surveillance and Health Equity Science of the American Cancer Society in Atlanta, Georgia, published their results as a research letter in JAMA Internal Medicine.
 

After Smoking Cessation

Overall, 11,860 cardiovascular, 10,935 cancer-related, and 2,060 respiratory-related deaths were considered from over 5 million patient years. Taken from these figures, the mortality risks of continuous smokers were 2.3 times (cardiovascular), 3.4 times (cancer-related), and 13.3 times (respiratory-related) higher than those of continuous nonsmokers.

Within 10 years of stopping smoking, the following occurred:

  • The cardiovascular mortality risk fell by 1.47 times, compared with nonsmokers (by 36% compared with smokers).
  • The cancer-related mortality risk fell by 2.13 times, compared with nonsmokers (by 47% compared with smokers).
  • The respiratory-related mortality risk fell by 6.35 times, compared with nonsmokers (by 43% compared with smokers).

In the second decade after stopping smoking, the risk dropped even further. The researchers observed the following trends:

  • The cardiovascular mortality risk fell by 1.26 times.
  • The cancer-related mortality risk fell by 1.59 times.
  • The respiratory-related mortality risk fell by 3.63 times — each time compared with nonsmokers.

During the third decade after stopping smoking, the risk continued to decrease. The trends were as follows:

  • The cardiovascular mortality risk fell by 1.07 times.
  • The cancer-related mortality risk fell by 1.34 times.
  • The respiratory-related mortality risk fell by 2.34 times, compared with nonsmokers.

30 Years Later

Only after more than 30 years of not smoking was the cardiovascular-related mortality risk 0.96 and, therefore, no longer significant. Compared with nonsmokers, the cancer-related mortality risk was 1.16, and the respiratory-related mortality risk was 1.31.

Therefore, former smokers can reduce their cardiovascular mortality risk by 100%, the cancer-related by 93%, and the respiratory-related mortality risk by 97%.

The result reinforces earlier analyses on the reduction in mortality risks by stopping smoking, with fewer participants. Smokers, therefore, benefit more the longer that they can refrain from using tobacco. “The earlier in life that smoking is given up, the better,” the authors wrote. But even in the first 10 years, the mortality risks examined decreased by a statistically significant 36% (cardiovascular) to 47% (cancer-related).
 

An Underestimation?

One disadvantage of the study is that the participants’ data were collected using personal questionnaires. For this reason, participants may have reported their tobacco consumption as being lower than it was, particularly because these questionnaires are often answered in hindsight, the authors pointed out.

In addition, some of the participants who reported stopping smoking completely may have only reduced their consumption. However, both circumstances would cause the results of the analysis to be even clearer, compared with reality, and therefore better.

This article was translated from the Medscape German edition.

A version of this article appeared on Medscape.com.

Publications
Publications
Topics
Article Type
Sections
Article Source

FROM JAMA INTERNAL MEDICINE

Disallow All Ads
Content Gating
No Gating (article Unlocked/Free)
Alternative CME
Disqus Comments
Default
Use ProPublica
Hide sidebar & use full width
render the right sidebar.
Conference Recap Checkbox
Not Conference Recap
Clinical Edge
Display the Slideshow in this Article
Medscape Article
Display survey writer
Reuters content
Disable Inline Native ads
WebMD Article

CPAP Oversells and Underperforms

Article Type
Changed
Tue, 01/02/2024 - 15:34

Continuous positive airway pressure (CPAP) is first-line therapy for sleep-related breathing disorders (SRBDs). Obstructive sleep apnea (OSA) is the major player in the SRBDs space, with a prevalence approaching 100% in adult men using current diagnostic criteria. Patients with OSA and comorbid cardiovascular disease (CVD) are diagnosed with OSA syndrome, and CPAP is prescribed. Primary care physicians and cardiologists are quick to refer patients with CVD to sleep docs to see whether CPAP can improve CVD-related outcomes.

What the Studies Show

There’s a problem though. CPAP doesn’t seem to improve CVD-related outcomes. In some cases, it’s even harmful. Let’s do a quick review. In 2005, the CANPAP study found CPAP didn’t improve a composite CVD outcome that included mortality. A post hoc analysis found that it actually increased mortality if central apneas weren’t eliminated. The post hoc analysis also found benefit when central apneas were eliminated, but for all-comers, CPAP didn’t improve outcomes. Strike one.

Enter adaptive servo-ventilation (ASV). If CANPAP showed that success depended on eliminating central apneas, why not use ASV for all patients with CVD and central apneas or Cheyne-Stokes respirations? ASV eliminates central apneas and Cheyne-Stokes. Well, that didn’t work either. The randomized, controlled SERVE-HF trial, published in 2015, showed that ASV increases all-cause and CVD-specific mortality. Oops. That’s two trials showing that CPAP and ASV can increase mortality in patients with heart failure. Strike two.

Alright. But that’s heart failure. What about hypertension or coronary artery disease (CAD)? Shouldn’t such patients be treated with CPAP to reduce CVD risk? After all, there’s all those surrogate outcomes data for CPAP — it improves vascular tone and lowers catecholamines and all that stuff. Doesn’t it lower blood pressure too? Surely CPAP benefits patients with CVD who don’t have heart failure, right?

Not really. The RICCADSA study, published in 2016, found that CPAP didn’t reduce a composite of CVD outcomes in patients with newly revascularized CAD. The SAVE trial published the same year had a similar design with similar results. CPAP did not improve CVD-related outcomes. Most recently, the ISAACC study was negative. That’s three negative randomized controlled trials in less than 5 years showing CPAP doesn’t affect CVD-related outcomes in high-risk populations with known disease. Strike three?

CPAP provides no benefit for CVD and possible harm when treating heart failure. Surely CPAP is useful for patients with hypertension. Let’s see. The American Academy of Sleep Medicine (AASM) conducted meta-analyses for the guideline it produced recommending CPAP for patients with comorbid hypertension. They note that 24-hour blood pressure measurements are best correlated with outcomes. CPAP did lead to significant 24-hour blood pressure reduction, but guess how large it was? For systolic blood pressure, it was 1.5 mm Hg; for diastolic pressure, it was 1.6 mm Hg. That’s it.

How did the AASM summarize and interpret the above data in their 2019 guidelines for prescribing CPAP? Although covered in their detailed review, both heart failure and CVD are left out of their primary recommendations . They do provide a conditional recommendation for prescribing CPAP to patients with comorbid hypertension that states, “The majority of well-informed patients would choose the intervention over no treatment.” Really? If you were told that CPAP provides less reduction in blood pressure than dietary changes and/or medications, would you choose to wear it or take a pill once a day? Remember, you have to take the pill anyway to get your blood pressure to target unless your pressure is only 1.5-1.6 mm Hg above normal. Where does one find patients who are anxious to wear a mask to bed for minimal benefit and a 20% copay? I’ve yet to meet one.

As always, the pressure pushers are undeterred by inconvenient evidence. A secondary analysis of adherent patients in RICCADSA resorts to the “bait and switch” that’s propped up CPAP enthusiasts for decades: Compare adherent patients versus those who are not (or those who refuse treatment) to prove benefit. The flaws to this approach are obvious. First, performing a post hoc analysis that reintroduces all of the confounding that plagues existing CPAP data negates the benefits of randomization, fancy statistics notwithstanding. Second, it belies the reality that in well-controlled, well-conducted randomized trials where patients get far more support than those in the community (and sometimes are preselected for adherence), a majority simply won’t use CPAP . Excluding the nonadherent or comparing them with the adherent is the epitome of selection bias.

The editorial accompanying the ISAACC study is a tour de force in CPAP apologies. The apnea-hypopnea index (AHI) isn’t the right metric — this one’s invoked often. Never mind that the very premise that OSA causes CVD is from observational data based on the AHI. If you abandon the AHI, don’t you lose your justification for prospective trials targeting CVD with CPAP?

Even better, in an argument fit for a Twitter ban, the author suggests that patients in ISAACC, SAVE, and RICCADSA couldn’t benefit because they already have CVD. The very concept, refuted by decades of secondary prevention research in cardiology, implies that CPAP should be used for primary prevention. Only a sleep researcher could spin a negative study into an expansion of CPAP indications. Others in the AASM have made similar proposals.

 

 

Final Thoughts

The sleep field lacks unblinded realists capable of choosing wisely. A little therapeutic underconfidence is warranted. Diseases and therapies will always have champions. Prudence and restraint? Not so much. The AASM could summarize the CPAP literature in a single recommendation: “If your patient is sleepy, CPAP might help them feel better if their disease is moderate or severe.” All other indications are soft.

A version of this article first appeared on Medscape.com.

Aaron B. Holley, MD, is a professor of medicine at Uniformed Services University in Bethesda, Maryland, and a pulmonary/sleep and critical care medicine physician at MedStar Washington Hospital Center in Washington, DC. He covers a  wide range of topics in pulmonary, critical care, and sleep medicine .

Publications
Topics
Sections

Continuous positive airway pressure (CPAP) is first-line therapy for sleep-related breathing disorders (SRBDs). Obstructive sleep apnea (OSA) is the major player in the SRBDs space, with a prevalence approaching 100% in adult men using current diagnostic criteria. Patients with OSA and comorbid cardiovascular disease (CVD) are diagnosed with OSA syndrome, and CPAP is prescribed. Primary care physicians and cardiologists are quick to refer patients with CVD to sleep docs to see whether CPAP can improve CVD-related outcomes.

What the Studies Show

There’s a problem though. CPAP doesn’t seem to improve CVD-related outcomes. In some cases, it’s even harmful. Let’s do a quick review. In 2005, the CANPAP study found CPAP didn’t improve a composite CVD outcome that included mortality. A post hoc analysis found that it actually increased mortality if central apneas weren’t eliminated. The post hoc analysis also found benefit when central apneas were eliminated, but for all-comers, CPAP didn’t improve outcomes. Strike one.

Enter adaptive servo-ventilation (ASV). If CANPAP showed that success depended on eliminating central apneas, why not use ASV for all patients with CVD and central apneas or Cheyne-Stokes respirations? ASV eliminates central apneas and Cheyne-Stokes. Well, that didn’t work either. The randomized, controlled SERVE-HF trial, published in 2015, showed that ASV increases all-cause and CVD-specific mortality. Oops. That’s two trials showing that CPAP and ASV can increase mortality in patients with heart failure. Strike two.

Alright. But that’s heart failure. What about hypertension or coronary artery disease (CAD)? Shouldn’t such patients be treated with CPAP to reduce CVD risk? After all, there’s all those surrogate outcomes data for CPAP — it improves vascular tone and lowers catecholamines and all that stuff. Doesn’t it lower blood pressure too? Surely CPAP benefits patients with CVD who don’t have heart failure, right?

Not really. The RICCADSA study, published in 2016, found that CPAP didn’t reduce a composite of CVD outcomes in patients with newly revascularized CAD. The SAVE trial published the same year had a similar design with similar results. CPAP did not improve CVD-related outcomes. Most recently, the ISAACC study was negative. That’s three negative randomized controlled trials in less than 5 years showing CPAP doesn’t affect CVD-related outcomes in high-risk populations with known disease. Strike three?

CPAP provides no benefit for CVD and possible harm when treating heart failure. Surely CPAP is useful for patients with hypertension. Let’s see. The American Academy of Sleep Medicine (AASM) conducted meta-analyses for the guideline it produced recommending CPAP for patients with comorbid hypertension. They note that 24-hour blood pressure measurements are best correlated with outcomes. CPAP did lead to significant 24-hour blood pressure reduction, but guess how large it was? For systolic blood pressure, it was 1.5 mm Hg; for diastolic pressure, it was 1.6 mm Hg. That’s it.

How did the AASM summarize and interpret the above data in their 2019 guidelines for prescribing CPAP? Although covered in their detailed review, both heart failure and CVD are left out of their primary recommendations . They do provide a conditional recommendation for prescribing CPAP to patients with comorbid hypertension that states, “The majority of well-informed patients would choose the intervention over no treatment.” Really? If you were told that CPAP provides less reduction in blood pressure than dietary changes and/or medications, would you choose to wear it or take a pill once a day? Remember, you have to take the pill anyway to get your blood pressure to target unless your pressure is only 1.5-1.6 mm Hg above normal. Where does one find patients who are anxious to wear a mask to bed for minimal benefit and a 20% copay? I’ve yet to meet one.

As always, the pressure pushers are undeterred by inconvenient evidence. A secondary analysis of adherent patients in RICCADSA resorts to the “bait and switch” that’s propped up CPAP enthusiasts for decades: Compare adherent patients versus those who are not (or those who refuse treatment) to prove benefit. The flaws to this approach are obvious. First, performing a post hoc analysis that reintroduces all of the confounding that plagues existing CPAP data negates the benefits of randomization, fancy statistics notwithstanding. Second, it belies the reality that in well-controlled, well-conducted randomized trials where patients get far more support than those in the community (and sometimes are preselected for adherence), a majority simply won’t use CPAP . Excluding the nonadherent or comparing them with the adherent is the epitome of selection bias.

The editorial accompanying the ISAACC study is a tour de force in CPAP apologies. The apnea-hypopnea index (AHI) isn’t the right metric — this one’s invoked often. Never mind that the very premise that OSA causes CVD is from observational data based on the AHI. If you abandon the AHI, don’t you lose your justification for prospective trials targeting CVD with CPAP?

Even better, in an argument fit for a Twitter ban, the author suggests that patients in ISAACC, SAVE, and RICCADSA couldn’t benefit because they already have CVD. The very concept, refuted by decades of secondary prevention research in cardiology, implies that CPAP should be used for primary prevention. Only a sleep researcher could spin a negative study into an expansion of CPAP indications. Others in the AASM have made similar proposals.

 

 

Final Thoughts

The sleep field lacks unblinded realists capable of choosing wisely. A little therapeutic underconfidence is warranted. Diseases and therapies will always have champions. Prudence and restraint? Not so much. The AASM could summarize the CPAP literature in a single recommendation: “If your patient is sleepy, CPAP might help them feel better if their disease is moderate or severe.” All other indications are soft.

A version of this article first appeared on Medscape.com.

Aaron B. Holley, MD, is a professor of medicine at Uniformed Services University in Bethesda, Maryland, and a pulmonary/sleep and critical care medicine physician at MedStar Washington Hospital Center in Washington, DC. He covers a  wide range of topics in pulmonary, critical care, and sleep medicine .

Continuous positive airway pressure (CPAP) is first-line therapy for sleep-related breathing disorders (SRBDs). Obstructive sleep apnea (OSA) is the major player in the SRBDs space, with a prevalence approaching 100% in adult men using current diagnostic criteria. Patients with OSA and comorbid cardiovascular disease (CVD) are diagnosed with OSA syndrome, and CPAP is prescribed. Primary care physicians and cardiologists are quick to refer patients with CVD to sleep docs to see whether CPAP can improve CVD-related outcomes.

What the Studies Show

There’s a problem though. CPAP doesn’t seem to improve CVD-related outcomes. In some cases, it’s even harmful. Let’s do a quick review. In 2005, the CANPAP study found CPAP didn’t improve a composite CVD outcome that included mortality. A post hoc analysis found that it actually increased mortality if central apneas weren’t eliminated. The post hoc analysis also found benefit when central apneas were eliminated, but for all-comers, CPAP didn’t improve outcomes. Strike one.

Enter adaptive servo-ventilation (ASV). If CANPAP showed that success depended on eliminating central apneas, why not use ASV for all patients with CVD and central apneas or Cheyne-Stokes respirations? ASV eliminates central apneas and Cheyne-Stokes. Well, that didn’t work either. The randomized, controlled SERVE-HF trial, published in 2015, showed that ASV increases all-cause and CVD-specific mortality. Oops. That’s two trials showing that CPAP and ASV can increase mortality in patients with heart failure. Strike two.

Alright. But that’s heart failure. What about hypertension or coronary artery disease (CAD)? Shouldn’t such patients be treated with CPAP to reduce CVD risk? After all, there’s all those surrogate outcomes data for CPAP — it improves vascular tone and lowers catecholamines and all that stuff. Doesn’t it lower blood pressure too? Surely CPAP benefits patients with CVD who don’t have heart failure, right?

Not really. The RICCADSA study, published in 2016, found that CPAP didn’t reduce a composite of CVD outcomes in patients with newly revascularized CAD. The SAVE trial published the same year had a similar design with similar results. CPAP did not improve CVD-related outcomes. Most recently, the ISAACC study was negative. That’s three negative randomized controlled trials in less than 5 years showing CPAP doesn’t affect CVD-related outcomes in high-risk populations with known disease. Strike three?

CPAP provides no benefit for CVD and possible harm when treating heart failure. Surely CPAP is useful for patients with hypertension. Let’s see. The American Academy of Sleep Medicine (AASM) conducted meta-analyses for the guideline it produced recommending CPAP for patients with comorbid hypertension. They note that 24-hour blood pressure measurements are best correlated with outcomes. CPAP did lead to significant 24-hour blood pressure reduction, but guess how large it was? For systolic blood pressure, it was 1.5 mm Hg; for diastolic pressure, it was 1.6 mm Hg. That’s it.

How did the AASM summarize and interpret the above data in their 2019 guidelines for prescribing CPAP? Although covered in their detailed review, both heart failure and CVD are left out of their primary recommendations . They do provide a conditional recommendation for prescribing CPAP to patients with comorbid hypertension that states, “The majority of well-informed patients would choose the intervention over no treatment.” Really? If you were told that CPAP provides less reduction in blood pressure than dietary changes and/or medications, would you choose to wear it or take a pill once a day? Remember, you have to take the pill anyway to get your blood pressure to target unless your pressure is only 1.5-1.6 mm Hg above normal. Where does one find patients who are anxious to wear a mask to bed for minimal benefit and a 20% copay? I’ve yet to meet one.

As always, the pressure pushers are undeterred by inconvenient evidence. A secondary analysis of adherent patients in RICCADSA resorts to the “bait and switch” that’s propped up CPAP enthusiasts for decades: Compare adherent patients versus those who are not (or those who refuse treatment) to prove benefit. The flaws to this approach are obvious. First, performing a post hoc analysis that reintroduces all of the confounding that plagues existing CPAP data negates the benefits of randomization, fancy statistics notwithstanding. Second, it belies the reality that in well-controlled, well-conducted randomized trials where patients get far more support than those in the community (and sometimes are preselected for adherence), a majority simply won’t use CPAP . Excluding the nonadherent or comparing them with the adherent is the epitome of selection bias.

The editorial accompanying the ISAACC study is a tour de force in CPAP apologies. The apnea-hypopnea index (AHI) isn’t the right metric — this one’s invoked often. Never mind that the very premise that OSA causes CVD is from observational data based on the AHI. If you abandon the AHI, don’t you lose your justification for prospective trials targeting CVD with CPAP?

Even better, in an argument fit for a Twitter ban, the author suggests that patients in ISAACC, SAVE, and RICCADSA couldn’t benefit because they already have CVD. The very concept, refuted by decades of secondary prevention research in cardiology, implies that CPAP should be used for primary prevention. Only a sleep researcher could spin a negative study into an expansion of CPAP indications. Others in the AASM have made similar proposals.

 

 

Final Thoughts

The sleep field lacks unblinded realists capable of choosing wisely. A little therapeutic underconfidence is warranted. Diseases and therapies will always have champions. Prudence and restraint? Not so much. The AASM could summarize the CPAP literature in a single recommendation: “If your patient is sleepy, CPAP might help them feel better if their disease is moderate or severe.” All other indications are soft.

A version of this article first appeared on Medscape.com.

Aaron B. Holley, MD, is a professor of medicine at Uniformed Services University in Bethesda, Maryland, and a pulmonary/sleep and critical care medicine physician at MedStar Washington Hospital Center in Washington, DC. He covers a  wide range of topics in pulmonary, critical care, and sleep medicine .

Publications
Publications
Topics
Article Type
Sections
Disallow All Ads
Content Gating
No Gating (article Unlocked/Free)
Alternative CME
Disqus Comments
Default
Use ProPublica
Hide sidebar & use full width
render the right sidebar.
Conference Recap Checkbox
Not Conference Recap
Clinical Edge
Display the Slideshow in this Article
Medscape Article
Display survey writer
Reuters content
Disable Inline Native ads
WebMD Article

COVID Strain JN.1 Is Now a ‘Variant of Interest,’ WHO Says

Article Type
Changed
Thu, 12/21/2023 - 14:31

The World Health Organization called the COVID-19 variant JN.1 a standalone “variant of interest” and said JN.1 will drive an increase in cases of the virus, the global health agency has announced.

JN.1 was previously grouped with its relative, BA.2.86, but has increased so much in the past 4 weeks that the WHO moved it to standalone status, according to a summary published by the agency. The prevalence of JN.1 worldwide jumped from 3% for the week ending November 5 to 27% for the week ending December 3. During that same period, JN.1 rose from 1% to 66% of cases in the Western Pacific, which stretches across 37 countries, from China and Mongolia to Australia and New Zealand.

In the United States, JN.1 has been increasing rapidly. The variant accounted for an estimated 21% of cases for the 2-week period ending December 9, up from 8% during the 2 weeks prior.

SARS-CoV-2 is the virus that causes COVID, and like other viruses, it evolves over time, sometimes changing how the virus affects people or how well existing treatments and vaccines work against it.

The WHO and CDC have said the current COVID vaccine appears to protect people against severe symptoms due to JN.1, and the WHO called the rising variant’s public health risk “low.”

“As we observe the rise of the JN.1 variant, it’s important to note that while it may be spreading more widely, there is currently no significant evidence suggesting it is more severe or that it poses a substantial public health risk,” John Brownstein, PhD, chief innovation officer at Boston Children’s Hospital, told ABC News.

In its recent risk analysis, the WHO did acknowledge that it’s not certain whether JN.1 has a higher risk of evading immunity or causing more severe symptoms than other strains. The WHO advised countries to further study how much JN.1 can evade existing antibodies and whether the variant results in more severe disease.

The latest CDC data show that 11% of COVID tests reported to the agency are positive, and 23,432 people were hospitalized with severe symptoms within a 7-day period. The CDC urgently asked people to get vaccinated against respiratory illnesses like the flu and COVID-19 ahead of the holidays as cases rise nationwide.

“Getting vaccinated now can help prevent hospitalizations and save lives,” the agency advised.


A version of this article originally appeared on WebMD.com.

Publications
Topics
Sections

The World Health Organization called the COVID-19 variant JN.1 a standalone “variant of interest” and said JN.1 will drive an increase in cases of the virus, the global health agency has announced.

JN.1 was previously grouped with its relative, BA.2.86, but has increased so much in the past 4 weeks that the WHO moved it to standalone status, according to a summary published by the agency. The prevalence of JN.1 worldwide jumped from 3% for the week ending November 5 to 27% for the week ending December 3. During that same period, JN.1 rose from 1% to 66% of cases in the Western Pacific, which stretches across 37 countries, from China and Mongolia to Australia and New Zealand.

In the United States, JN.1 has been increasing rapidly. The variant accounted for an estimated 21% of cases for the 2-week period ending December 9, up from 8% during the 2 weeks prior.

SARS-CoV-2 is the virus that causes COVID, and like other viruses, it evolves over time, sometimes changing how the virus affects people or how well existing treatments and vaccines work against it.

The WHO and CDC have said the current COVID vaccine appears to protect people against severe symptoms due to JN.1, and the WHO called the rising variant’s public health risk “low.”

“As we observe the rise of the JN.1 variant, it’s important to note that while it may be spreading more widely, there is currently no significant evidence suggesting it is more severe or that it poses a substantial public health risk,” John Brownstein, PhD, chief innovation officer at Boston Children’s Hospital, told ABC News.

In its recent risk analysis, the WHO did acknowledge that it’s not certain whether JN.1 has a higher risk of evading immunity or causing more severe symptoms than other strains. The WHO advised countries to further study how much JN.1 can evade existing antibodies and whether the variant results in more severe disease.

The latest CDC data show that 11% of COVID tests reported to the agency are positive, and 23,432 people were hospitalized with severe symptoms within a 7-day period. The CDC urgently asked people to get vaccinated against respiratory illnesses like the flu and COVID-19 ahead of the holidays as cases rise nationwide.

“Getting vaccinated now can help prevent hospitalizations and save lives,” the agency advised.


A version of this article originally appeared on WebMD.com.

The World Health Organization called the COVID-19 variant JN.1 a standalone “variant of interest” and said JN.1 will drive an increase in cases of the virus, the global health agency has announced.

JN.1 was previously grouped with its relative, BA.2.86, but has increased so much in the past 4 weeks that the WHO moved it to standalone status, according to a summary published by the agency. The prevalence of JN.1 worldwide jumped from 3% for the week ending November 5 to 27% for the week ending December 3. During that same period, JN.1 rose from 1% to 66% of cases in the Western Pacific, which stretches across 37 countries, from China and Mongolia to Australia and New Zealand.

In the United States, JN.1 has been increasing rapidly. The variant accounted for an estimated 21% of cases for the 2-week period ending December 9, up from 8% during the 2 weeks prior.

SARS-CoV-2 is the virus that causes COVID, and like other viruses, it evolves over time, sometimes changing how the virus affects people or how well existing treatments and vaccines work against it.

The WHO and CDC have said the current COVID vaccine appears to protect people against severe symptoms due to JN.1, and the WHO called the rising variant’s public health risk “low.”

“As we observe the rise of the JN.1 variant, it’s important to note that while it may be spreading more widely, there is currently no significant evidence suggesting it is more severe or that it poses a substantial public health risk,” John Brownstein, PhD, chief innovation officer at Boston Children’s Hospital, told ABC News.

In its recent risk analysis, the WHO did acknowledge that it’s not certain whether JN.1 has a higher risk of evading immunity or causing more severe symptoms than other strains. The WHO advised countries to further study how much JN.1 can evade existing antibodies and whether the variant results in more severe disease.

The latest CDC data show that 11% of COVID tests reported to the agency are positive, and 23,432 people were hospitalized with severe symptoms within a 7-day period. The CDC urgently asked people to get vaccinated against respiratory illnesses like the flu and COVID-19 ahead of the holidays as cases rise nationwide.

“Getting vaccinated now can help prevent hospitalizations and save lives,” the agency advised.


A version of this article originally appeared on WebMD.com.

Publications
Publications
Topics
Article Type
Sections
Disallow All Ads
Content Gating
No Gating (article Unlocked/Free)
Alternative CME
Disqus Comments
Default
Use ProPublica
Hide sidebar & use full width
render the right sidebar.
Conference Recap Checkbox
Not Conference Recap
Clinical Edge
Display the Slideshow in this Article
Medscape Article
Display survey writer
Reuters content
Disable Inline Native ads
WebMD Article

Systemic Bias in AI Models May Undermine Diagnostic Accuracy

Article Type
Changed
Wed, 12/20/2023 - 13:15

Systematically biased artificial intelligence (AI) models did not improve clinicians’ accuracy in diagnosing hospitalized patients, based on data from more than 450 clinicians.

“Artificial Intelligence (AI) could support clinicians in their diagnostic decisions of hospitalized patients but could also be biased and cause potential harm,” said Sarah Jabbour, MSE, a PhD candidate in computer science and engineering at the University of Michigan, Ann Arbor, in an interview.

“Regulatory guidance has suggested that the use of AI explanations could mitigate these harms, but the effectiveness of using AI explanations has not been established,” she said.

To examine whether AI explanations can be effective in mitigating the potential harms of systemic bias in AI models, Ms. Jabbour and colleagues conducted a randomized clinical vignette survey study. The survey was administered between April 2022 and January 2023 across 13 states, and the study population included hospitalist physicians, nurse practitioners, and physician assistants. The results were published in JAMA.

Participants were randomized to AI predictions with AI explanations (226 clinicians) or without AI explanations (231 clinicians).

The primary outcome was diagnostic accuracy for pneumonia, heart failure, and chronic obstructive pulmonary disease, defined as the number of correct diagnoses over the total number of assessments, the researchers wrote.

The clinicians viewed nine clinical vignettes of patients hospitalized with acute respiratory failure, including their presenting symptoms, physical examination, laboratory results, and chest radiographs. Clinicians viewed two vignettes with no AI model input to establish baseline diagnostic accuracy. They made three assessments in each vignette, one for each diagnosis. The order of the vignettes was two without AI predictions (to establish baseline diagnostic accuracy), six with AI predictions, and one with a clinical consultation by a hypothetical colleague. The vignettes included standard and systematically biased AI models.

The baseline diagnostic accuracy was 73% for the diagnoses of pneumonia, heart failure, and chronic obstructive pulmonary disease. Clinicians’ accuracy increased by 2.9% when they viewed a standard diagnostic AI model without explanations and by 4.4% when they viewed models with AI explanations.

However, clinicians’ accuracy decreased by 11.3% after viewing systematically biased AI model predictions without explanations compared with baseline, and biased AI model predictions with explanations decreased accuracy by 9.1%.

The decrease in accuracy with systematically biased AI predictions without explanations was mainly attributable to a decrease in the participants’ diagnostic specificity, the researchers noted, but the addition of explanations did little to improve it, the researchers said.

Potentially Useful but Still Imperfect

The findings were limited by several factors including the use of a web-based survey, which differs from surveys in a clinical setting, the researchers wrote. Other limitations included the younger than average study population, and the focus on the clinicians making treatment decisions, vs other clinicians who might have a better understanding of the AI explanations.

“In our study, explanations were presented in a way that were considered to be obvious, where the AI model was completely focused on areas of the chest X-rays unrelated to the clinical condition,” Ms. Jabbour told this news organization. “We hypothesized that if presented with such explanations, the participants in our study would notice that the model was behaving incorrectly and not rely on its predictions. This was surprisingly not the case, and the explanations when presented alongside biased AI predictions had seemingly no effect in mitigating clinicians’ overreliance on biased AI,” she said.

“AI is being developed at an extraordinary rate, and our study shows that it has the potential to improve clinical decision-making. At the same time, it could harm clinical decision-making when biased,” Ms. Jabbour said. “We must be thoughtful about how to carefully integrate AI into clinical workflows, with the goal of improving clinical care while not introducing systematic errors or harming patients,” she added.

Looking ahead, “There are several potential research areas that could be explored,” said Ms. Jabbour. “Researchers should focus on careful validation of AI models to identify biased model behavior prior to deployment. AI researchers should also continue including and communicating with clinicians during the development of AI tools to better understand clinicians’ needs and how they interact with AI,” she said. “This is not an exhaustive list of research directions, and it will take much discussion between experts across disciplines such as AI, human computer interaction, and medicine to ultimately deploy AI safely into clinical care.”

 

 

Dont Overestimate AI

“With the increasing use of artificial intelligence and machine learning in other spheres, there has been an increase in interest in exploring how they can be utilized to improve clinical outcomes,” said Suman Pal, MD, assistant professor in the division of hospital medicine at the University of New Mexico, Albuquerque, in an interview. “However, concerns remain regarding the possible harms and ways to mitigate them,” said Dr. Pal, who was not involved in the current study.

In the current study, “It was interesting to note that explanations did not significantly mitigate the decrease in clinician accuracy from systematically biased AI model predictions,” Dr. Pal said.

“For the clinician, the findings of this study caution against overreliance on AI in clinical decision-making, especially because of the risk of exacerbating existing health disparities due to systemic inequities in existing literature,” Dr. Pal told this news organization.

“Additional research is needed to explore how clinicians can be better trained in identifying both the utility and the limitations of AI and into methods of validation and continuous quality checks with integration of AI into clinical workflows,” he noted.

The study was funded by the National Heart, Lung, and Blood Institute. The researchers had no financial conflicts to disclose. Dr. Pal had no financial conflicts to disclose.

A version of this article first appeared on Medscape.com.

Publications
Topics
Sections

Systematically biased artificial intelligence (AI) models did not improve clinicians’ accuracy in diagnosing hospitalized patients, based on data from more than 450 clinicians.

“Artificial Intelligence (AI) could support clinicians in their diagnostic decisions of hospitalized patients but could also be biased and cause potential harm,” said Sarah Jabbour, MSE, a PhD candidate in computer science and engineering at the University of Michigan, Ann Arbor, in an interview.

“Regulatory guidance has suggested that the use of AI explanations could mitigate these harms, but the effectiveness of using AI explanations has not been established,” she said.

To examine whether AI explanations can be effective in mitigating the potential harms of systemic bias in AI models, Ms. Jabbour and colleagues conducted a randomized clinical vignette survey study. The survey was administered between April 2022 and January 2023 across 13 states, and the study population included hospitalist physicians, nurse practitioners, and physician assistants. The results were published in JAMA.

Participants were randomized to AI predictions with AI explanations (226 clinicians) or without AI explanations (231 clinicians).

The primary outcome was diagnostic accuracy for pneumonia, heart failure, and chronic obstructive pulmonary disease, defined as the number of correct diagnoses over the total number of assessments, the researchers wrote.

The clinicians viewed nine clinical vignettes of patients hospitalized with acute respiratory failure, including their presenting symptoms, physical examination, laboratory results, and chest radiographs. Clinicians viewed two vignettes with no AI model input to establish baseline diagnostic accuracy. They made three assessments in each vignette, one for each diagnosis. The order of the vignettes was two without AI predictions (to establish baseline diagnostic accuracy), six with AI predictions, and one with a clinical consultation by a hypothetical colleague. The vignettes included standard and systematically biased AI models.

The baseline diagnostic accuracy was 73% for the diagnoses of pneumonia, heart failure, and chronic obstructive pulmonary disease. Clinicians’ accuracy increased by 2.9% when they viewed a standard diagnostic AI model without explanations and by 4.4% when they viewed models with AI explanations.

However, clinicians’ accuracy decreased by 11.3% after viewing systematically biased AI model predictions without explanations compared with baseline, and biased AI model predictions with explanations decreased accuracy by 9.1%.

The decrease in accuracy with systematically biased AI predictions without explanations was mainly attributable to a decrease in the participants’ diagnostic specificity, the researchers noted, but the addition of explanations did little to improve it, the researchers said.

Potentially Useful but Still Imperfect

The findings were limited by several factors including the use of a web-based survey, which differs from surveys in a clinical setting, the researchers wrote. Other limitations included the younger than average study population, and the focus on the clinicians making treatment decisions, vs other clinicians who might have a better understanding of the AI explanations.

“In our study, explanations were presented in a way that were considered to be obvious, where the AI model was completely focused on areas of the chest X-rays unrelated to the clinical condition,” Ms. Jabbour told this news organization. “We hypothesized that if presented with such explanations, the participants in our study would notice that the model was behaving incorrectly and not rely on its predictions. This was surprisingly not the case, and the explanations when presented alongside biased AI predictions had seemingly no effect in mitigating clinicians’ overreliance on biased AI,” she said.

“AI is being developed at an extraordinary rate, and our study shows that it has the potential to improve clinical decision-making. At the same time, it could harm clinical decision-making when biased,” Ms. Jabbour said. “We must be thoughtful about how to carefully integrate AI into clinical workflows, with the goal of improving clinical care while not introducing systematic errors or harming patients,” she added.

Looking ahead, “There are several potential research areas that could be explored,” said Ms. Jabbour. “Researchers should focus on careful validation of AI models to identify biased model behavior prior to deployment. AI researchers should also continue including and communicating with clinicians during the development of AI tools to better understand clinicians’ needs and how they interact with AI,” she said. “This is not an exhaustive list of research directions, and it will take much discussion between experts across disciplines such as AI, human computer interaction, and medicine to ultimately deploy AI safely into clinical care.”

 

 

Dont Overestimate AI

“With the increasing use of artificial intelligence and machine learning in other spheres, there has been an increase in interest in exploring how they can be utilized to improve clinical outcomes,” said Suman Pal, MD, assistant professor in the division of hospital medicine at the University of New Mexico, Albuquerque, in an interview. “However, concerns remain regarding the possible harms and ways to mitigate them,” said Dr. Pal, who was not involved in the current study.

In the current study, “It was interesting to note that explanations did not significantly mitigate the decrease in clinician accuracy from systematically biased AI model predictions,” Dr. Pal said.

“For the clinician, the findings of this study caution against overreliance on AI in clinical decision-making, especially because of the risk of exacerbating existing health disparities due to systemic inequities in existing literature,” Dr. Pal told this news organization.

“Additional research is needed to explore how clinicians can be better trained in identifying both the utility and the limitations of AI and into methods of validation and continuous quality checks with integration of AI into clinical workflows,” he noted.

The study was funded by the National Heart, Lung, and Blood Institute. The researchers had no financial conflicts to disclose. Dr. Pal had no financial conflicts to disclose.

A version of this article first appeared on Medscape.com.

Systematically biased artificial intelligence (AI) models did not improve clinicians’ accuracy in diagnosing hospitalized patients, based on data from more than 450 clinicians.

“Artificial Intelligence (AI) could support clinicians in their diagnostic decisions of hospitalized patients but could also be biased and cause potential harm,” said Sarah Jabbour, MSE, a PhD candidate in computer science and engineering at the University of Michigan, Ann Arbor, in an interview.

“Regulatory guidance has suggested that the use of AI explanations could mitigate these harms, but the effectiveness of using AI explanations has not been established,” she said.

To examine whether AI explanations can be effective in mitigating the potential harms of systemic bias in AI models, Ms. Jabbour and colleagues conducted a randomized clinical vignette survey study. The survey was administered between April 2022 and January 2023 across 13 states, and the study population included hospitalist physicians, nurse practitioners, and physician assistants. The results were published in JAMA.

Participants were randomized to AI predictions with AI explanations (226 clinicians) or without AI explanations (231 clinicians).

The primary outcome was diagnostic accuracy for pneumonia, heart failure, and chronic obstructive pulmonary disease, defined as the number of correct diagnoses over the total number of assessments, the researchers wrote.

The clinicians viewed nine clinical vignettes of patients hospitalized with acute respiratory failure, including their presenting symptoms, physical examination, laboratory results, and chest radiographs. Clinicians viewed two vignettes with no AI model input to establish baseline diagnostic accuracy. They made three assessments in each vignette, one for each diagnosis. The order of the vignettes was two without AI predictions (to establish baseline diagnostic accuracy), six with AI predictions, and one with a clinical consultation by a hypothetical colleague. The vignettes included standard and systematically biased AI models.

The baseline diagnostic accuracy was 73% for the diagnoses of pneumonia, heart failure, and chronic obstructive pulmonary disease. Clinicians’ accuracy increased by 2.9% when they viewed a standard diagnostic AI model without explanations and by 4.4% when they viewed models with AI explanations.

However, clinicians’ accuracy decreased by 11.3% after viewing systematically biased AI model predictions without explanations compared with baseline, and biased AI model predictions with explanations decreased accuracy by 9.1%.

The decrease in accuracy with systematically biased AI predictions without explanations was mainly attributable to a decrease in the participants’ diagnostic specificity, the researchers noted, but the addition of explanations did little to improve it, the researchers said.

Potentially Useful but Still Imperfect

The findings were limited by several factors including the use of a web-based survey, which differs from surveys in a clinical setting, the researchers wrote. Other limitations included the younger than average study population, and the focus on the clinicians making treatment decisions, vs other clinicians who might have a better understanding of the AI explanations.

“In our study, explanations were presented in a way that were considered to be obvious, where the AI model was completely focused on areas of the chest X-rays unrelated to the clinical condition,” Ms. Jabbour told this news organization. “We hypothesized that if presented with such explanations, the participants in our study would notice that the model was behaving incorrectly and not rely on its predictions. This was surprisingly not the case, and the explanations when presented alongside biased AI predictions had seemingly no effect in mitigating clinicians’ overreliance on biased AI,” she said.

“AI is being developed at an extraordinary rate, and our study shows that it has the potential to improve clinical decision-making. At the same time, it could harm clinical decision-making when biased,” Ms. Jabbour said. “We must be thoughtful about how to carefully integrate AI into clinical workflows, with the goal of improving clinical care while not introducing systematic errors or harming patients,” she added.

Looking ahead, “There are several potential research areas that could be explored,” said Ms. Jabbour. “Researchers should focus on careful validation of AI models to identify biased model behavior prior to deployment. AI researchers should also continue including and communicating with clinicians during the development of AI tools to better understand clinicians’ needs and how they interact with AI,” she said. “This is not an exhaustive list of research directions, and it will take much discussion between experts across disciplines such as AI, human computer interaction, and medicine to ultimately deploy AI safely into clinical care.”

 

 

Dont Overestimate AI

“With the increasing use of artificial intelligence and machine learning in other spheres, there has been an increase in interest in exploring how they can be utilized to improve clinical outcomes,” said Suman Pal, MD, assistant professor in the division of hospital medicine at the University of New Mexico, Albuquerque, in an interview. “However, concerns remain regarding the possible harms and ways to mitigate them,” said Dr. Pal, who was not involved in the current study.

In the current study, “It was interesting to note that explanations did not significantly mitigate the decrease in clinician accuracy from systematically biased AI model predictions,” Dr. Pal said.

“For the clinician, the findings of this study caution against overreliance on AI in clinical decision-making, especially because of the risk of exacerbating existing health disparities due to systemic inequities in existing literature,” Dr. Pal told this news organization.

“Additional research is needed to explore how clinicians can be better trained in identifying both the utility and the limitations of AI and into methods of validation and continuous quality checks with integration of AI into clinical workflows,” he noted.

The study was funded by the National Heart, Lung, and Blood Institute. The researchers had no financial conflicts to disclose. Dr. Pal had no financial conflicts to disclose.

A version of this article first appeared on Medscape.com.

Publications
Publications
Topics
Article Type
Sections
Article Source

FROM JAMA

Disallow All Ads
Content Gating
No Gating (article Unlocked/Free)
Alternative CME
Disqus Comments
Default
Use ProPublica
Hide sidebar & use full width
render the right sidebar.
Conference Recap Checkbox
Not Conference Recap
Clinical Edge
Display the Slideshow in this Article
Medscape Article
Display survey writer
Reuters content
Disable Inline Native ads
WebMD Article

Blood Eosinophil-Directed Prednisolone Proves Noninferior to Standard Care for COPD Exacerbations

Article Type
Changed
Tue, 12/19/2023 - 15:14

 

TOPLINE:

Both blood eosinophil-directed treatment (BET) and standard care treatment (ST) similarly reduced treatment failure following acute exacerbations in chronic obstructive pulmonary disease (COPD).

METHODOLOGY:

  • The researchers randomized 152 adults with a mean age of 70 years to BET or a placebo (if eosinophil counts were less than 2%) or to standard care treatment regardless of baseline eosinophil counts; the final population available for analysis included 47 patients in the blood eosinophil group and 46 in the primary care group, with 73 and 71 exacerbations, respectively.
  • Participants were assessed at baseline and at day 14, day 30, and day 90 after exacerbation; the primary outcome was the rate of treatment failure at 30 days post exacerbation, defined as any need for retreatment with antibiotics or steroids, hospitalization, or death; secondary outcomes included health-related quality of life, forced expiratory volume in 1 second, and visual analogue score respiratory symptoms.
  • Participants were recruited from 14 general practices between November 6, 2017, and April 30, 2020; the study was terminated on April 30, 2023, because of the COVID-19 pandemic.

TAKEAWAY:

  • BET was noninferior to ST in a noninferiority analysis.
  • At 30 days post exacerbation, 14 treatment failures had occurred in the BET group and 23 in the ST group; the relative risk was 0.60 (P = .070).
  • The frequency of adverse events was similar between the groups; the most common adverse events were glycosuria and hospital admission for COPD exacerbation (2% in the BET group and 1% in the ST group for both event types), and no deaths occurred during the study period.
  • Subgroup analysis showed the greatest benefit in both groups was to patients with higher eosinophil counts who received prednisolone.

IN PRACTICE: 

“There was improvement of lung function, quality of life, and symptoms in exacerbations with low eosinophil count independent of whether placebo or prednisolone was prescribed,” the authors wrote in their discussion.

SOURCE:

The lead author on the study was Sanjay Ramakrishnan, MBBS, University of Oxford, United Kingdom. The study was published online in Lancet Respiratory Medicine .

LIMITATIONS:

A key limitation was an error in the randomization code that prevented the trial’s completion as a superiority study; other limitations included the relatively low number of exacerbations associated with low eosinophil counts and reduction in the recommended length of treatment with prednisolone during the study period.

DISCLOSURES:

The study was supported by the National Institute for Health and Care Research. Dr. Ramakrishnan disclosed personal salary support from the National Institute for Health and Care Research, an unrestricted research grant from AstraZeneca to his institution, and speaker fees and conference travel support from AstraZeneca, all unrelated to the current study.

A version of this article first appeared on Medscape.com.

Publications
Topics
Sections

 

TOPLINE:

Both blood eosinophil-directed treatment (BET) and standard care treatment (ST) similarly reduced treatment failure following acute exacerbations in chronic obstructive pulmonary disease (COPD).

METHODOLOGY:

  • The researchers randomized 152 adults with a mean age of 70 years to BET or a placebo (if eosinophil counts were less than 2%) or to standard care treatment regardless of baseline eosinophil counts; the final population available for analysis included 47 patients in the blood eosinophil group and 46 in the primary care group, with 73 and 71 exacerbations, respectively.
  • Participants were assessed at baseline and at day 14, day 30, and day 90 after exacerbation; the primary outcome was the rate of treatment failure at 30 days post exacerbation, defined as any need for retreatment with antibiotics or steroids, hospitalization, or death; secondary outcomes included health-related quality of life, forced expiratory volume in 1 second, and visual analogue score respiratory symptoms.
  • Participants were recruited from 14 general practices between November 6, 2017, and April 30, 2020; the study was terminated on April 30, 2023, because of the COVID-19 pandemic.

TAKEAWAY:

  • BET was noninferior to ST in a noninferiority analysis.
  • At 30 days post exacerbation, 14 treatment failures had occurred in the BET group and 23 in the ST group; the relative risk was 0.60 (P = .070).
  • The frequency of adverse events was similar between the groups; the most common adverse events were glycosuria and hospital admission for COPD exacerbation (2% in the BET group and 1% in the ST group for both event types), and no deaths occurred during the study period.
  • Subgroup analysis showed the greatest benefit in both groups was to patients with higher eosinophil counts who received prednisolone.

IN PRACTICE: 

“There was improvement of lung function, quality of life, and symptoms in exacerbations with low eosinophil count independent of whether placebo or prednisolone was prescribed,” the authors wrote in their discussion.

SOURCE:

The lead author on the study was Sanjay Ramakrishnan, MBBS, University of Oxford, United Kingdom. The study was published online in Lancet Respiratory Medicine .

LIMITATIONS:

A key limitation was an error in the randomization code that prevented the trial’s completion as a superiority study; other limitations included the relatively low number of exacerbations associated with low eosinophil counts and reduction in the recommended length of treatment with prednisolone during the study period.

DISCLOSURES:

The study was supported by the National Institute for Health and Care Research. Dr. Ramakrishnan disclosed personal salary support from the National Institute for Health and Care Research, an unrestricted research grant from AstraZeneca to his institution, and speaker fees and conference travel support from AstraZeneca, all unrelated to the current study.

A version of this article first appeared on Medscape.com.

 

TOPLINE:

Both blood eosinophil-directed treatment (BET) and standard care treatment (ST) similarly reduced treatment failure following acute exacerbations in chronic obstructive pulmonary disease (COPD).

METHODOLOGY:

  • The researchers randomized 152 adults with a mean age of 70 years to BET or a placebo (if eosinophil counts were less than 2%) or to standard care treatment regardless of baseline eosinophil counts; the final population available for analysis included 47 patients in the blood eosinophil group and 46 in the primary care group, with 73 and 71 exacerbations, respectively.
  • Participants were assessed at baseline and at day 14, day 30, and day 90 after exacerbation; the primary outcome was the rate of treatment failure at 30 days post exacerbation, defined as any need for retreatment with antibiotics or steroids, hospitalization, or death; secondary outcomes included health-related quality of life, forced expiratory volume in 1 second, and visual analogue score respiratory symptoms.
  • Participants were recruited from 14 general practices between November 6, 2017, and April 30, 2020; the study was terminated on April 30, 2023, because of the COVID-19 pandemic.

TAKEAWAY:

  • BET was noninferior to ST in a noninferiority analysis.
  • At 30 days post exacerbation, 14 treatment failures had occurred in the BET group and 23 in the ST group; the relative risk was 0.60 (P = .070).
  • The frequency of adverse events was similar between the groups; the most common adverse events were glycosuria and hospital admission for COPD exacerbation (2% in the BET group and 1% in the ST group for both event types), and no deaths occurred during the study period.
  • Subgroup analysis showed the greatest benefit in both groups was to patients with higher eosinophil counts who received prednisolone.

IN PRACTICE: 

“There was improvement of lung function, quality of life, and symptoms in exacerbations with low eosinophil count independent of whether placebo or prednisolone was prescribed,” the authors wrote in their discussion.

SOURCE:

The lead author on the study was Sanjay Ramakrishnan, MBBS, University of Oxford, United Kingdom. The study was published online in Lancet Respiratory Medicine .

LIMITATIONS:

A key limitation was an error in the randomization code that prevented the trial’s completion as a superiority study; other limitations included the relatively low number of exacerbations associated with low eosinophil counts and reduction in the recommended length of treatment with prednisolone during the study period.

DISCLOSURES:

The study was supported by the National Institute for Health and Care Research. Dr. Ramakrishnan disclosed personal salary support from the National Institute for Health and Care Research, an unrestricted research grant from AstraZeneca to his institution, and speaker fees and conference travel support from AstraZeneca, all unrelated to the current study.

A version of this article first appeared on Medscape.com.

Publications
Publications
Topics
Article Type
Sections
Disallow All Ads
Content Gating
No Gating (article Unlocked/Free)
Alternative CME
Disqus Comments
Default
Use ProPublica
Hide sidebar & use full width
render the right sidebar.
Conference Recap Checkbox
Not Conference Recap
Clinical Edge
Display the Slideshow in this Article
Medscape Article
Display survey writer
Reuters content
Disable Inline Native ads
WebMD Article